Вы находитесь на странице: 1из 68

Total Marks : 200

TEST - 10(MOCK TEST - 8) Mark Scored : 0

1 Geospatial technology is being used by the National Remote Sensing Centre (NRSC) to support
National Mission for Clean Ganga (NMCG) in which of the following ways?
1. Water quality monitoring
2. Urban sprawl change mapping
3. Non-point source pollution assessment
4. Comprehensive basin planning

Select the correct answer using the codes below.


A. 1 and 4 only
B. 2 and 3 only
C. 1, 2 and 4 only
D. 1, 2, 3 and 4

Your Answer :
Correct Answer : D

7
Answer Justification :
2 27
0 3
Justification: Statement 3: Non-point pollution means that pollution 1 is 5
not being generated from a
9 4
- a case, it is difficult to estimate
single point and the pollutants are dispersed location-wise. In such
pollution load by conventional means. Geospatial data showsm a map of all such non-point sources.
i l .co
Learning: NRSC is a part of Indian Space Research m aOrganization (ISRO).
7 @g
i
Some of the tasks enlisted by NRSCsraas part of support to NMCG are generation of
k
comprehensive GIS database, i tmwater quality assessment using satellite data of main Ganga
p ul real time water quality data visualization.
from Kannauj to Varanasi,
a -
isr
t M
It also includes high quality multispectral satellite image, aerial topographical survey, urban
k i
l change mapping, non-point source pollution assessment etc.
sprawl
Pu
A holistic approach is being adopted by NMCG to keep river Ganga clean by identifying five
km. stretches from the edge of the bank/flood plain to the nearest main road for a
comprehensive planning approach.

Q Source: Samir/jk (Release ID :171223)

2 Consider the following statements. Assertion (A): Inflation redistributes wealth from creditors to
debtors. Reason (R): Higher inflation lowers the effective cost of credit for the debtor.

In the context of the above, which of these is correct?


A. A is correct, and R is an appropriate explanation of A.
B. A is correct, but R is not an appropriate explanation of A.
C. A is correct, but R is incorrect.
D. Both A and R are incorrect.
www.insightsias.com 1
© Insights Active Learning | All rights reserved - 78168. You may not reproduce, distribute or exploit the contents in any form without
written permission by copyright owner. Copyright infringers may face civil and criminal liability
Total Marks : 200
TEST - 10(MOCK TEST - 8) Mark Scored : 0

Your Answer :
Correct Answer : A

Answer Justification :

Justification: Suppose a bank lent you Rs. 1 lakh at 10% p.a. interest rate.

When the inflation rate is 6%, the bank is getting a net interest rate of 4% (ten minus six). This net
interest rate is the bank’s profit margin from lending this sum of money.

When inflation increases, to say 8%, bank’s net earning reduces as net interest rate (or real interest
rate) has come down to 2%.

This benefits the debtor because the real value of Rs. 1 Lakh has reduced due to inflation, and in
effect he is paying less than what he borrowed.

Q Source: Chapter 7: Indian Economy – Ramesh Singh


7
2 27
3
3 As per Washington Consensus
1 50
in4a growing economy.
1. There should be a greater role of state vis-à-vis the private sector 9
-
2. Developed countries should stop deregulation measures which,
o m in the past, have led to increased
economic inequality. l. c i
m a
Which of the above is/are correct? @g
A. 1 only
s r a7
B. 2 only m i
t
C. Both 1 and 2
u lki
D. None -p
i sra
Your Answer : M
l k it : D
Correct Answer
Pu
Answer Justification :

Justification: It is considered to constitute the "standard" reform package promoted for crisis-
wracked developing countries by Washington, D.C.–based institutions such as the International
Monetary Fund (IMF), World Bank, and the US Treasury Department.

It basically refers to a more general orientation towards a strongly market-based approach such as
macroeconomic stabilization, economic opening with respect to both trade and investment, and the
expansion of market forces within the domestic economy.

Q Source: Revision: Indian Economy – Ramesh Singh

4 Under the India International Science Festival(IISF), 2017, the participating countries in the Science
and Technology Ministers conclave include
1. Bangladesh
2. USA

www.insightsias.com 2
© Insights Active Learning | All rights reserved - 78168. You may not reproduce, distribute or exploit the contents in any form without
written permission by copyright owner. Copyright infringers may face civil and criminal liability
Total Marks : 200
TEST - 10(MOCK TEST - 8) Mark Scored : 0

3. China
4. Nepal

Select the correct answer using the codes below.


A. 2 and 3 only
B. 1 and 4 only
C. 1, 2 and 3 only
D. 1, 2, 3 and 4

Your Answer :
Correct Answer : B

Answer Justification :

Justification: IISF, 2017 is to be held in Chennai. It is S&T Partnership for Addressing Societal
Challenges on a bilateral and multilateral level.
7
Participants: S&T Ministers of Bangladesh, Afghanistan, Nepal & Portugal 2 27
0 3
5
The focus will be: 41
-9
. com
ai l
capacity building through mechanisms such as fellowships, internships, training, data sharing,
m
@g
joint projects and technology transfer.
7
is ra
Co-develop and deploy affordable
i t m and innovative technologies that provide solution to societal
p ulk
problems will also be a common agenda for the Conclave.

a -
is
Q Source: As mentioned
r above
t M
u lki
P recently seen in news, is related to which of the aspects of Gandhiji’s life?
5 Kirti Mandir,

A. It is located in the birth place of Gandhi as a memorial temple.


B. It was the first ashram built in Gujarat by Gandhi.
C. This was the place where Gandhi gave his first discourse.
D. For the first time when Gandhiji was held as a political prisoner, he returned to this temple
after his release.

Your Answer :
Correct Answer : A

Answer Justification :

Learning: It is located in Porbandar, the birth place of Mahatma Gandhi today

Kirti Mandir is the memorial temple built in memory of Mohandas Karamchand Gandhi and
Kasturba Gandhi.

www.insightsias.com 3
© Insights Active Learning | All rights reserved - 78168. You may not reproduce, distribute or exploit the contents in any form without
written permission by copyright owner. Copyright infringers may face civil and criminal liability
Total Marks : 200
TEST - 10(MOCK TEST - 8) Mark Scored : 0

At the time when Gandhi was released for the last time in year 1944 from the Aga Khan Palace by
the British Government, the residential public of Porbandar had decided to construct an ideal
memorial on the birth place of Gandhi.

There are life size oil paintings of Mahatma Gandhi and Kasturba kept side by side in the centre of
Kirti Mandir.

Q Source: AKT/NT (Release ID :171320)

6 An economy heats up due to


1. Demand deficit
2. Excess supply
3. High liquidity infusion

Select the correct answer using the codes below.


A. 1 and 2 only 7
B. 2 only
2 27
C. 3 only 0 3
5
D. 2 and 3 only 41
-9
Your Answer :
. com
Correct Answer : C ai l
gm
7 @
sra
Answer Justification :

m i
k t
Justification: Overheating of anieconomy occurs when its productive capacity is unable to keep
pace with growing aggregateu l
demand leading to inflation.
- p
a
srdemand and low supply, high consumption and low savings can lead to a
When there is excess
i
situation whereM
there is a shortage of investible capital.
l k it
Pu is unable to supply demanded goods and prices start increases.
The economy

Q Source: Chapter 7 – Section B – Business Cycle: Indian Economy – Ramesh Singh

7 Article 340 of the constitution was recently in news. Consider the following with reference to it.
1. It provides for appointment of a Commission by the President to investigate the conditions of
women and religious minorities.
2. The action taken in pursuance to this article must be reported to the Parliament by the President.

Which of the above is/are correct?


A. 1 only
B. 2 only
C. Both 1 and 2
D. None

Your Answer :

www.insightsias.com 4
© Insights Active Learning | All rights reserved - 78168. You may not reproduce, distribute or exploit the contents in any form without
written permission by copyright owner. Copyright infringers may face civil and criminal liability
Total Marks : 200
TEST - 10(MOCK TEST - 8) Mark Scored : 0

Correct Answer : B

Answer Justification :

Justification: Statement 1: It is to investigate the conditions of socially and educationally backward


classes.

It is intended to improve their condition and to decide upon the grants that should be made for the
purpose.

The latest commission (2017) was appointed to Examine Sub-Categorization of other Backward
Classes under Article 340 of the Constitution.

Statement 2: The President shall cause a copy of the report so presented together with a
memorandum explaining the action taken thereon to be laid before each House of Parliament.

Q Source: As mentioned above


7
2 27
0 3
8 A seller’s market is an economic situation in which 5
41
-9
A. Goods are scarce and sellers can keep prices high m
oleading
B. i l
There is only one buyer and a large number of seller. c to monopsony.
C. m a
Goods are plenty and sellers are forced to keep prices low
D.
g
@ of buyer leading to monopoly.
There is only one seller and a large number
7
isra
Your Answer :
i t m
Correct Answer : A
p ulk
a -
sr :
Answer Justification
i
M
it is a market situation wherein sellers are outnumbered by buyers by a big margin and
Learning:khis
l
Pu of goods available for sale becomes far less than the quantity demanded.
the quantity

During boom as demand outstrips supply, the situation can often be seen across.

For e.g. in summer, there is a huge demand for mangoes. In a certain location where there are just
two mango sellers and the demand is extremely high for the same, then that would be a seller's
market.

Since the demand is high, the sellers will be able to charge a high price for mangoes and get better
sale conditions.

Q Source: Chapter 7 – Section B – Business Cycle: Indian Economy – Ramesh Singh

9 The Bay of Fundy in Canada is known for


1. Integrating largest volume of freshwater
2. Most indented coastline
3. Largest frequency of cyclones

www.insightsias.com 5
© Insights Active Learning | All rights reserved - 78168. You may not reproduce, distribute or exploit the contents in any form without
written permission by copyright owner. Copyright infringers may face civil and criminal liability
Total Marks : 200
TEST - 10(MOCK TEST - 8) Mark Scored : 0

Select the correct answer using the codes below.


A. 1 only
B. 2 and 3 only
C. 3 only
D. None of the above

Your Answer :
Correct Answer : D

Answer Justification :

Learning: It has the highest tidal range in the world.

In the Bay of Fundy they occur in Nova Scotia, Canada.

The tidal bulge is 15 - 16 m. Because there are two high tides and two low tides every day (roughly
7 estimate, the
a 24 hour period); then a tide must come in within about a six hour period. As a rough
7
tide rises about 240 cm an hour (1,440 cm divided by 6 hours). 2 2
5 03
If you have walked down a beach with a steep cliff alongside (which4 is1common there), make sure
you watch the tides. - 9
m
l
If you walk for about an hour and then notice that the itide .cois coming in, the water will be over your
head before you get back to where you started! gm
a
a 7@
s
Q Source: Page 122: Chapter 14: Fundamentals
i r of Physical Geography: 11th NCERT

k i tm
ul
10 With reference to the EnergypEfficiency Services Limited (EESL), consider the following statements.
-
i sraNational
1. It is a joint venture of Grid Network (NGN) and Public Energy Service Company (PESC).

t M
2. It was established with the sole objective of providing and generating finance for energy efficiency
projects. ki
ulthe resource centre for capacity building of State DISCOMs.
3. It actsPas
4. It is owned on a public-private partnership (PPP) basis.

Select the correct answer using the codes below.


A. 1 and 3 only
B. 1, 2 and 4 only
C. 3 only
D. 2, 3 and 4 only

Your Answer :
Correct Answer : C

Answer Justification :

Justification & Learning: It is a joint venture of NTPC Limited, Power Finance Corporation, Rural
Electrification Corporation and POWERGRID set up under Ministry of Power to facilitate
implementation of energy efficiency projects.

www.insightsias.com 6
© Insights Active Learning | All rights reserved - 78168. You may not reproduce, distribute or exploit the contents in any form without
written permission by copyright owner. Copyright infringers may face civil and criminal liability
Total Marks : 200
TEST - 10(MOCK TEST - 8) Mark Scored : 0

EESL seeks to unlock energy efficiency market in India, estimated to at US$12 billion that can
potentially result in energy savings of up to 20 per cent of current consumption, by way of
innovative business and implementation models.

It is the largest public Super Energy Service Company (ESCO) in the world.

EESL has been able to successfully aggregate demand and has undertaken bulk distribution of LED
bulbs that has led to more than 80% reduction of costs.

Q Source: RM/VM/AS: (Release ID :171263): EESL to procure 10,000 Electric Vehicles from TATA
Motors

11 An inflationary gap signals that the economy is in which part of the trade cycle?

A. Boom
B. Recession
7
C. Depression
2 27
D. Recovery 0 3
5
41
Your Answer : -9
Correct Answer : A
. com
ai l
Answer Justification : gm
7 @
sra are being used over their capacity, factories are
Learning: It is a situation where resources
i
m wage rates increase because labour is used beyond normal
operating with increasing averagetcosts;
i
hours at overtime pay rates.ul k
- p
The inflationary gap a
sris so named because the relative increase in real GDP causes an economy to
M i
increase its consumption, which causes prices to rise in the long run.
l k it
Pu cause of the gap is considered to be expansionary monetary policies carried out by the
The main
government.

Q Source: Chapter 7: Indian Economy – Ramesh Singh

12 Consumer Price Index (CPI) Inflation in India is measured on a

A. Weekly basis
B. Quarterly basis
C. Monthly basis
D. Yearly basis

Your Answer :
Correct Answer : C

Answer Justification :

www.insightsias.com 7
© Insights Active Learning | All rights reserved - 78168. You may not reproduce, distribute or exploit the contents in any form without
written permission by copyright owner. Copyright infringers may face civil and criminal liability
Total Marks : 200
TEST - 10(MOCK TEST - 8) Mark Scored : 0

Justification: Reference dates for calculating inflation are from the preceding and succeeding
month of the same year.

You can see the data here https://data.gov.in/catalog/all-india-consumer-price-index-ruralurban

Until recently, India had several sectoral consumer price indices (CPI) and a national level
wholesale price index (WPI). A national level consumer price index (CPI-combined) was released in
2011 and the inflation target for monetary policy is prescribed in term of all-India CPI-combined
index.

Q Source: Chapter 7: Indian Economy – Ramesh Singh

13 Phytoplankton are generally found in upper layers of ocean water due to


1. Absence of nutrients in deep ocean water
2. Sunlight abundance in top layers
3. Calm top layers of water due to absence of mixing between top and bottom layers
7
2 27
Select the correct answer using the codes below. 0 3
5
A. 3 only
41
B. 1 and 2 only -9
C. 2 only
. com
D. 2 and 3 only
ai l
gm
Your Answer : 7 @
Correct Answer : C isra
i t m
Answer Justification :
p ulk
a -
i sr
Justification: Phytoplankton in the open ocean need both sunlight and nutrients (such as nitrate
and phosphate)M to be able to photosynthesize.
l k it
Pu
Sunlight is only available in the uppermost layers.

During photosynthesis, the nutrients are quickly used up by phytoplankton so they are not
available for long periods in the upper layers under normal circumstances.

This is indeed the case in tropical waters, and as a result they are very unproductive.

To escape this problem the seawater needs to be mixed regularly to bring the nutrient rich
deep waters up to the sunlight zone where the phytoplankton can grow.

This is one of the reasons why areas where cold and warm currents meet are very productive; the
collision of currents causes mixing.

www.insightsias.com 8
© Insights Active Learning | All rights reserved - 78168. You may not reproduce, distribute or exploit the contents in any form without
written permission by copyright owner. Copyright infringers may face civil and criminal liability
Total Marks : 200
TEST - 10(MOCK TEST - 8) Mark Scored : 0

Q Source: Page 124: Fundamentals of Physical Geography: 11th NCERT

14 Consider the following with reference to the Wholesale Price Index (WPI).
1. The present WPI base is 2004-05.
2. It excludes consumer durables and precious metals.
3. Highest weightage has been accorded to manufactured products in the index.

Select the correct answer using the codes below.


A. 1 and 2 only
B. 3 only
C. 2 only
D. None of the above

Your Answer :
Correct Answer : B
7
Answer Justification : 2 27
3
1 50
94 Fuel & Power and
Justification: There are three major groups in WPI – Primary Articles,
-
Manufacture Products. Highest weightage has been accorded to the manufactured products.
. com
ai l
Consumer durables are a part of the manufactured products category.
g m
The WPI series base year, article composition7 @and weightage has been revised to 2011-12 recently.
r a
m is the work relating to revision of the existing series of WPI.
The Office of the Economic Adviser
l k it takes

- puEconomy – Ramesh Singh


Q Source: Chapter 7: Indian

i sra
it MGlobal Entrepreneurship Summit is
15 The theme of 2017
k
l
Pu
A. “Women First, Prosperity for All”
B. “Nature with Man”
C. “Make sustainable, break sustainable”
D. “Strengths and Diversity”

Your Answer :
Correct Answer : A

Answer Justification :

Learning: Women represent tremendous promise for economic growth and prosperity -- but in both
developing and developed countries, also face tremendous barriers to building businesses.

NITI Aayog is taking the lead in organising this Summit for the Government of India based on ths
theme.

This would be a unique opportunity for Indian Startups and Innovators to interact and network with
www.insightsias.com 9
© Insights Active Learning | All rights reserved - 78168. You may not reproduce, distribute or exploit the contents in any form without
written permission by copyright owner. Copyright infringers may face civil and criminal liability
Total Marks : 200
TEST - 10(MOCK TEST - 8) Mark Scored : 0

the finest entrepreneurs of the world.

The GES is the preeminent annual gathering of emerging entrepreneurs, investors, and business
leaders from around the world.

Q Source: AKT/SH/VK (Release ID :171339)

16 Which of these forces help in the movement or generation of ocean currents?


1. Coriolis force
2. Gravitational force
3. Solar insolation
4. Movement of wind

Select the correct answer using the codes below.


A. 2 and 3 only
B. 1 and 4 only 7
C. 2, 3 and 4 only
2 27
D. 1, 2, 3 and 4 0 3
5
41
Your Answer : -9
Correct Answer : D
. com
ai l
Answer Justification : gm
@
a7
sr currents are generated largely by wind. Their patterns
i
Justification: Statement 1 and 4: Surface
m
are determined by wind direction,
k i tCoriolis forces from the Earth’s rotation, and the position of
landforms that interact withu l currents.
the
- p
Statement 2: Near s the a
r equator the ocean water is about 8 cm higher in level than in the middle
i
M a very slight gradient and water tends to flow down the slope.
latitudes. This causes
l k it
Pu 3: Solar energy makes current warm or cold and decides their circulatory patterns.
Statement

Q Source: Page 123: Fundamentals of Physical Geography: 11th NCERT

17 Indicative planning is most likely to be followed by which of these economies?

A. Capitalist economies
B. Mixed economies
C. Communist economies
D. Primary economies

Your Answer :
Correct Answer : B

Answer Justification :

www.insightsias.com 10
© Insights Active Learning | All rights reserved - 78168. You may not reproduce, distribute or exploit the contents in any form without
written permission by copyright owner. Copyright infringers may face civil and criminal liability
Total Marks : 200
TEST - 10(MOCK TEST - 8) Mark Scored : 0

Justification: Option A: Purely Capitalist economies would generally not follow any planning and
let the market decide how much and when to produce.

Option B and C: In communist economies the planning is imperative not indicative in nature. So,
mixed economies are fit for indicative planning where a plan making body (from the State) sets
broad numerical targets to be achieved with the help of market forces.

Q Source: Revision: Indian Economy – Ramesh Singh

18 The Phillips Curve suggests that


1. Lower the inflation, lower is the unemployment
2. Higher the environmental degradation, higher is the economic inequality

Which of the above is/are correct?


A. 1 only
B. 2 only 7
C. Both 1 and 2
2 27
D. None 0 3
5
41
Your Answer : -9
Correct Answer : D
. com
ai l
Answer Justification : gm
@
r a7
isdiscuss
Justification: Statement 2: It does not about inequality or environmental degradation.

k i tm
ulthere is a ‘trade off’ between inflation and unemployment i.e. an
Statement 1: As per the curve
p
- them.
inverse relationship between
a
i sr
M that lower the inflation, higher the unemployment and higher the inflation,
The curve suggests
t
k i
lower thelunemployment.
P u
During 1960s, this idea was among the most important theories of the modern economists.
However, economists later contested this curve and suggested some modifications to it.

Q Source: Chapter 7: Indian Economy – Ramesh Singh

19 Which of the following measures will increase inflation rather than taming it?
1. Taking measures to curb hoarding of essential items
2. Tight monetary policy
3. Import of goods which are in short-supply
4. Loose fiscal policy

Select the correct answer using the codes below.


A. 1, 3 and 4 only
B. 1, 2 and 3 only
C. 4 only

www.insightsias.com 11
© Insights Active Learning | All rights reserved - 78168. You may not reproduce, distribute or exploit the contents in any form without
written permission by copyright owner. Copyright infringers may face civil and criminal liability
Total Marks : 200
TEST - 10(MOCK TEST - 8) Mark Scored : 0

D. 2 and 3 only

Your Answer :
Correct Answer : C

Answer Justification :

Justification: Statement 1 and 3: It is common in the case of onion, pulses etc where government
often cracks down on illegal hoarding to increase market supply, and at times imports pulses to
increase their domestic supply so that prices can be reduced.

Statement 2: Tight monetary policy is basically intended to cut down the money supply in the
economy by siphoning out the extra money from the economy, so that effective demand is reduced.

This is a short-term measure. In the long-run, the best way is to increase production with the help of
the best production practices.

7 7
Statement 4: Loose fiscal policy only increases the liquidity and pushes demand2thereby raising
3 2
inflation. 0 5
1
Q Source: Chapter 7: Indian Economy – Ramesh Singh - 94
. com
a i l
20 Consider the following about Global Wildlife Program (GWP).
m
@g (GEF).
1. It was launched by the Global Environment Facility
7
2. It is a World-Bank led global partnership.ra
3. It promotes wildlife conservation bym is
combatting illicit trafficking in wildlife.
4. India will host the GWP for thelk i t
year 2017.
p u
a -
i sr using the codes below.
Select the correct answer
M
A. 2 and 3 only
B. 1 and k 4tonly
i
l
C. P2uand 4 only
D. 1, 2, 3 and 4

Your Answer :
Correct Answer : D

Answer Justification :

Justification: To respond to the growing crisis and international call for action, the GEF in 2015
launched the “Global Partnership on Wildlife Conservation and Crime Prevention for Sustainable
Development” program also known as the GWP.

The GWP is a World-Bank led global partnership that promotes wildlife conservation and
sustainable development by combatting illicit trafficking in wildlife.

This seven-year, $131 million grant program is expected to leverage an additional $704 million in
additional co-financing from a wide range of partners to promote investments across Africa and

www.insightsias.com 12
© Insights Active Learning | All rights reserved - 78168. You may not reproduce, distribute or exploit the contents in any form without
written permission by copyright owner. Copyright infringers may face civil and criminal liability
Total Marks : 200
TEST - 10(MOCK TEST - 8) Mark Scored : 0

Asia.

Q Source:
http://www.downtoearth.org.in/news/india-to-host-global-wildlife-programme-from-october-2-58778

21 A shortage of investible capital in an economy can result from


1. Very high and erratic inflation
2. Low savings
3. High borrowing costs

Select the correct answer using the codes below.


A. 1 and 2 only
B. 2 and 3 only
C. 3 only
D. 1, 2 and 3
7
Your Answer : 2 27
0 3
Correct Answer : D 5
41
Answer Justification : -9
. com
Justification: Statement 1: High and erratic inflation l
aideters investors from making investment
decisions because real returns on investments are m
g uncertain.
7 @
sra of the money that is borrowed for infrastructure building
Statement 2 and 3: Savings forms the ibase
tm demand of capital lead to high borrowing costs and deter
in the economy. Low savings andihigh
k
investors.
p ul
-
Q Source: Chapters7ra – Section B – Business Cycle: Indian Economy – Ramesh Singh
M i
l k it
22 Among P u following, the state with the highest copping intensity is?
the

A. Andhra Pradesh
B. Maharashtra
C. Madhya Pradesh
D. Tamil Nadu

Your Answer :
Correct Answer : C

Answer Justification :

Concept: Cropping intensity shows how effectively the net sown area was utilized within a cropping
season. If the net sown area was left empty throughout the season, cropping intensity would be
zero.

If it was occupied fully once, it would be hundred per cent. If there are more than one cropping

www.insightsias.com 13
© Insights Active Learning | All rights reserved - 78168. You may not reproduce, distribute or exploit the contents in any form without
written permission by copyright owner. Copyright infringers may face civil and criminal liability
Total Marks : 200
TEST - 10(MOCK TEST - 8) Mark Scored : 0

seasons and the same land was sown multiple times, cropping intensity would be more than
hundred per cent.

Learning: Andhra Pradesh Cropping Intensity: 122.79 %

Madhya Pradesh: 150.66%

Maharashtra: 126.12 %

Tamil Nadu: 113.11%

Large cropping intensity does not mean that net sown area will be large as well.

Q Source: http://farmer.gov.in/agriland.aspx

23 Consider the following schemes of the Government of India.


1. SYPOG scheme aims at making young scientists aware of the important flagship 7 7
programmes of the
Government. 2 2
03technological
2. Under Science Village Scheme, each MP will adopt one village and fund5its
1
upgradation by way of MPLADS funds. 94 -
. com
Which of the above is/are correct?
a i l
A. 1 only
m
B. 2 only
7 @g
C. Both 1 and 2
isra
D. None
i t m
Your Answer : p ulk
a -
Correct Answer : A
i sr
M
kit
Answer Justification
l :
Pu
Justification: Statement 1: These schemes are aimed towards Sensitizing students and youth to
Flagship Programs of Government (SYPOG).

This covers schemes such as Swachh Bharat Abhiyan, Swastha Bharat, Digital India, Make in India,
Bio-diversity etc.

Statement 2: Under Science Village: Students and teachers will be nominated by Members of
Parliament from their adopted villages under Pradhan Manthri Sansad Aadarsh Gram Yojna.

Moreover, as a unique programme of IISF 2017, a Women Scientist & Entrepreneur’s Conclave
titled “Game Changers Driving Science for New India” is being organized to promote science and
entrepreneurship among the young women.

Q Source: IISF 2017

24 Consider the following statements with reference to the Consumer Price Index (CPI) data before the unification of the
CPI.

www.insightsias.com 14
© Insights Active Learning | All rights reserved - 78168. You may not reproduce, distribute or exploit the contents in any form without
written permission by copyright owner. Copyright infringers may face civil and criminal liability
Total Marks : 200
TEST - 10(MOCK TEST - 8) Mark Scored : 0

1. When the Pay Commissions recommended pay revisions, the base was the CPI Industrial Workers
(IW).
2. CPI for Agricultural Labourers (AL) was used for revising minimum wages for agricultural labourers
in different states.

Which of the above is/are correct?


A. 1 only
B. 2 only
C. Both 1 and 2
D. None

Your Answer :
Correct Answer : C

Answer Justification :

77 banks’ and
Justification: Statement 1: CPI-IW specifies the government employees (other than
2
2
3 are revised on the
embassies’ personnel). The wages/salaries of the central government employees
50is announced twice a year.
basis of the changes occurring in this index, the dearness allowance (DA)
1
4
-9
CPI for the Urban Non-Manual Employees (CPI-UNME) is basically used for determining dearness
m
ooperating
l .
allowances (DAs) of employees of some foreign companies c in India (i.e. airlines,
i financial services).
communications, banking, insurance, embassies, andaother
gm
@
Statement 2: The governments at the centre
s r a7and states remain vigilant regarding the changes in
CPI-AL as it shows the price impact oni the most vulnerable segment of the society, this segment
itmincome on the purchase of food articles.
spends almost 75 per cent of itsktotal
p ul
- Economy – Ramesh Singh
Q Source: Chapter 7: Indian
a
isr
M
25 Consider thel k it
following statements about Headline inflation.
P u
1. It is presently measured in terms of Wholesale Price Index (PPI) in India.
2. It excludes the prices of food and fuel that constitute the volatile component of inflation.

Which of the above is/are correct?


A. 1 only
B. 2 only
C. Both 1 and 2
D. None

Your Answer :
Correct Answer : D

Answer Justification :

Justification: Statement 1: RBI Governor had adopted the new Consumer Price Index (CPI)
(combined) as the key measure of inflation in 2014 based on the Urijeet Patel committee
recommendation.

www.insightsias.com 15
© Insights Active Learning | All rights reserved - 78168. You may not reproduce, distribute or exploit the contents in any form without
written permission by copyright owner. Copyright infringers may face civil and criminal liability
Total Marks : 200
TEST - 10(MOCK TEST - 8) Mark Scored : 0

Earlier, RBI had given more weightage to Wholesale Price Index (WPI) than CPI as the key measure
of inflation for all policy purposes.

Statement 2: Headline inflation is a measure of the total inflation within an economy, including
commodities such as food and energy prices (e.g., oil and gas). Core inflation calculations excludes
food and energy which tend to be much more volatile and prone to inflationary spikes.

Q Source: Chapter 7: Indian Economy – Ramesh Singh

26 Consider the following statements. Assertion (A): Irrigation intensity reduces consistently as one
moves from North to South in India. Reason (R): Deccan region is known for its hard rock structure
as compared to the plains of India.

In the context of the above, which of these is correct?


A. A is correct, and R is an appropriate explanation of A.
B. A is correct, but R is not an appropriate explanation of A.
7
C. A is incorrect, but R is correct.
2 27
D. A is correct, but R is incorrect. 0 3
5
41
Your Answer : -9
Correct Answer : C
. com
ai l
Answer Justification : gm
7 @
Justification: Hard rock structure does a setting up irrigation structures more difficult, but
srmake
i
irrigation intensity does not reduce
k i tm
in the fashion as explained by statement 1. It is a bit complex

p ul
and depends on a number of factors such as availability of river catchments, rock structure,
dispersal of agricultural -land, state investment in irrigation etc.
i sra
Mshows the irrigation intensity in India.
The figure below
t
lk i
P u

www.insightsias.com 16
© Insights Active Learning | All rights reserved - 78168. You may not reproduce, distribute or exploit the contents in any form without
written permission by copyright owner. Copyright infringers may face civil and criminal liability
Total Marks : 200
TEST - 10(MOCK TEST - 8) Mark Scored : 0

7
2 27
0 3
5
Q Source: Surprise questions 41
-9
om
c Interbank Local Currency Credit Line
l.the
27 The Union Cabinet has given its approval to the signing iof
a
Agreement and Cooperation Memorandum Relatingm to Credit Ratings by Exim Bank with
g
7@
participating member banks under BRICS Interbank Cooperation Mechanism. Consider the following
about it. a
sr member nations.
i
1. These are binding umbrella pacts on
k i tmconclude any individual contracts with related within the
BRICS
l
2. It allows Exim bank to negotiate and
framework of this pact. pu
-
ra by financial member-institutions would be possible by this pact.
3. Lending in single currency
is
M
Select thek
l it answer using the codes below.
correct
A. P1uonly
B. 2 and 3 only
C. 1 and 2 only
D. 1, 2 and 3

Your Answer :
Correct Answer : B

Answer Justification :

Justification: Statement 1 and 2: As both the Agreement and the MoU are umbrella pacts, and are
non-binding in nature, Exim Bank, leveraging this umbrella agreement, could enter into bilateral
agreement with any of these member institutions to raise resources for its business.

Signing of the Agreement will position Exim Bank in the international platform along with large
development finance institutions, like CDS, VEB and BNDES.

www.insightsias.com 17
© Insights Active Learning | All rights reserved - 78168. You may not reproduce, distribute or exploit the contents in any form without
written permission by copyright owner. Copyright infringers may face civil and criminal liability
Total Marks : 200
TEST - 10(MOCK TEST - 8) Mark Scored : 0

Statement 3: Lending in borrowing in multiple currencies creates the problems of valuation of the
debt as currency values fluctuates in the forex market. This will be avoided by single currency
lending.

As and when an opportunity arises for co-financing in commercial terms, by any two-member
institutions (say India and South Africa), lending in single currency by both the institutions would
also be possible.

Q Source: AKT/VBA/SH (Release ID :171174)

28 Consider the following about the ‘Saubhagya’ scheme that intends to provide energy access to all.
1. The scheme applies to rural households alone because the electricity development in urban areas is
managed by the municipalities.
2. The scheme would provide free connections and free electricity to all poor rural households.

Which of the above is/are correct? 7


A. 1 only
2 27
B. 2 only 0 3
5
C. Both 1 and 2 41
D. None -9
. com
Your Answer : ai l
Correct Answer : D gm
7 @
Answer Justification : isra
it m
Justification: Statement 1: u k
Itlfocuses on last mile connectivity and electricity connections to all
p
remaining un-electrified -households in rural as well as urban areas to achieve universal household
a
isrcountry.
electrification in the
M
it
lk2: Poor households would be provided electricity connections free of cost. Other
Statement
u
P would also be provided electricity connections under the scheme on payment of Rs.500
households
only.

There is no provision in the scheme to provide free power to any category of consumers. The cost of
electricity consumption shall have to be paid by the respective consumers as per prevailing tariff of
the DISCOM/Power Deptt.

Q Source: RM/AS (Release ID :171148)

29 The Wholesale Price Index (WPI) is released by

A. Office of the Governor, RBI


B. Ministry of Finance
C. Central Statistical Organization (CSO), Ministry of Statistics and Programme Implementation
D. Office of the Economic Adviser, DIPP

www.insightsias.com 18
© Insights Active Learning | All rights reserved - 78168. You may not reproduce, distribute or exploit the contents in any form without
written permission by copyright owner. Copyright infringers may face civil and criminal liability
Total Marks : 200
TEST - 10(MOCK TEST - 8) Mark Scored : 0

Your Answer :
Correct Answer : D

Answer Justification :

Learning: WPI is an important statistical indicator, as various policy decisions of the Government,
like inflation management, monitoring of prices of essential commodities etc., are based on it.

It is one of the key variables for monetary policy changes by the Reserve Bank of India. In addition
to its role as a policy variable, WPI is also used by various departments for arriving at the escalation
costs of various contracts.

Q Source: Chapter 7: Indian Economy – Ramesh Singh

30 The President has recently made which of the following appointments to the office of Governor in
different states?
7
1. B. D. Mishra as the Governor of Arunachal Pradesh
2 27
2. Satya Pal Malik as the Governor of Madhya Pradesh
5 03
3. Banwarilal Purohit as the Governor of Tamil Nadu
9 41
4. Jagdish Mukhi as the Governor of Nagaland -
om
Select the correct answer using the codes below.
a il.c
A. 1 and 2 only
gm
B. 3 and 4 only
a 7@
C. 1 and 3 only
is r
D. 2 and 4 only it m
u lk
Your Answer :
a -p
isr
Correct Answer : C
M
lk it
Answer Justification :
Pu
Justification: Jagdish Mukhi as the Governor of Assam. Satya Pal Malik as the Governor of Bihar.
Ganga Prasad as the Governor of Meghalaya.

Devendra Kumar Joshi, as the Lieutenant Governor of Andaman and Nicobar Islands.

The above appointments will take effect from the dates the incumbents assume charge of their
respective offices.

Q Source: AKT/SH (Release ID :171275)

31 The government has recently given its approval for implementation of umbrella scheme of
"Modernisation of Police Forces (MPF)" for years 2017-18 to 2019-20. Consider the following about
it.
1. It will be directed by the Centre, but financed entirely by the state governments.
2. Under the scheme, special sums are to be allotted for the internal security related expenditure of
Jammu & Kashmir and Left Wing Extremism (LWE) affected states.

www.insightsias.com 19
© Insights Active Learning | All rights reserved - 78168. You may not reproduce, distribute or exploit the contents in any form without
written permission by copyright owner. Copyright infringers may face civil and criminal liability
Total Marks : 200
TEST - 10(MOCK TEST - 8) Mark Scored : 0

3. The upgradation of investigation responsibilities of the police has been separated from the scheme
and its financing has been handed over to private sector agencies.

Select the correct answer using the codes below.


A. 2 only
B. 2 and 3 only
C. 1 and 2 only
D. 1 and 3 only

Your Answer :
Correct Answer : A

Answer Justification :

Justification: Statement 1: The financial outlay for the scheme over the three years period is Rs.
25,060 crore, out of which the Central Government share will be Rs. 18,636 crore and the States’
share will be Rs.6,424 crore. So, 1 is wrong. 77 2
2
5 03
Under the scheme special provisions have been made for internal
9 41 security, law and order,
women security, availability of modern weapons, mobility -of police forces, logistics support,
m Satellite Network, CCTNS
oNational
hiring of helicopters, upgradation of police wireless,
l . c
project, E-prison project etc. aim
@ g
a7
sr received special assistances in view of the impending
i
J&K, NE states and LWE states have
security situation. itmlk
u
-p
i sra also provides for setting up of a State-of Art forensic science laboratory
The umbrella scheme
M Andhra Pradesh etc.
in Amravati,
l k it
Pu
Q Source: As mentioned above

32 The National Register 2017 launched by the Airports Authority of India (AAI) contains
1. Details of all Civil and military Airports and Airstrips in the country
2. List of no-fly passengers and routes including those on private and joint venture airports

Which of the above is/are correct?


A. 1 only
B. 2 only
C. Both 1 and 2
D. None

Your Answer :
Correct Answer : D

www.insightsias.com 20
© Insights Active Learning | All rights reserved - 78168. You may not reproduce, distribute or exploit the contents in any form without
written permission by copyright owner. Copyright infringers may face civil and criminal liability
Total Marks : 200
TEST - 10(MOCK TEST - 8) Mark Scored : 0

Answer Justification :

Justification: Statement 1: National Register 2017 is a public document and it cannot contain
details of military airports and airstrips as it would threaten the security of these belts.

The National Register 2017 contains details pertaining to Airports and Airstrips owned by State
Governments, Private Organisations and Joint Venture Airports in addition to the airports owned/
managed by AAI.

Statement 2: There is no such provision.

Learning: The details and data will be valuable for planning of development of airports especially
under Regional Connectivity Scheme (RCS).

The details of 288 airports and airstrips could be of use for emergency landing, disaster
management, Defence and security related activities.

7
Q Source: NP/MS (Release ID :171182)
2 27
3
1 50
4 of the four great Kings of
33 Which scholar described the Emperor of the Rashtrakuta dynasty as9one
the World in the 9th century? -
. com
A. Traveller Marco Polo ai l
B. Francois Bernier who visited Mughal Indiag
m
a 7@
C. Arab scholar Sulaiman
is r
D. Persian scholar Al-Biruni
it m
l k
Your Answer :
- pu
Correct Answer : C ra
M is
k it
Answer Justification
l :
P u
Learning: Attracted by the grandiose of the Rashtrakutas, many scholars have attempted to write
on them.

Sulaiman was impressed by the state of arts, architecture, economy and military conquests of the
Rashtrakutas and hence described them so.

Al-Masudi has also written on limitary conquests of the Rashtrakutas as the 9th Century was a
melting pot of many military expeditions between Pratiharas, Palas, Rashtrakutas etc.

Q Source: Surprise questions

34 With reference to the Pt Deen Dayal Upadhayay Vigyan Gram Sankul Pariyojana, consider the
following:
1. Clusters of villages will be selected under the scheme for transforming them with the help of S&T
interventions.
2. The scheme exhorts local communities to migrate in for acquiring skilled jobs so that they can

www.insightsias.com 21
© Insights Active Learning | All rights reserved - 78168. You may not reproduce, distribute or exploit the contents in any form without
written permission by copyright owner. Copyright infringers may face civil and criminal liability
Total Marks : 200
TEST - 10(MOCK TEST - 8) Mark Scored : 0

return and contribute to the management of technological interventions in the villages.


3. It covers development of manufacturing and services sector exclusively, as agricultural sector
modernization would overlap with other schemes.

Select the correct answer using the codes below.


A. 1 only
B. 2 and 3 only
C. 1 and 3 only
D. 1, 2 and 3

Your Answer :
Correct Answer : A

Answer Justification :

Justification: Under the scheme, the department of science and technology has conceived to adopt
77 in a time
a few clusters of villages in Uttarakhand and transform them to become self-sustainable
2
bound manner through the tools of Science and Technology (S&T). 3 2
5 0
9 41
-
The key deliverable in this approach is to utilise local resources and locally available skill sets
o m
i l .c
and convert them in a manner using science and technology, that substantial value addition
takes place in their local produce and servicesa which can sustain the rural population locally.
gm
@
a7
sr clusters would be processing and value addition of
i
Areas of interventions in these selected
i
milk, honey, mushroom, herbal
k tmtea, forest produce, horticulture and local crops, medicinal &
ul
aromatic plants and traditional
p
craft and handloom of Uttarakhand.

a -
M isr
Practice of agriculture, agro-based cottage industries and animal husbandry in an eco-friendly
k it
l
manner will be emphasized during the implementation of the project.
Pu
Q Source: As mentioned above

35 The government has taken several initiatives in recent times to bring useful information through the
help of modern information technology to the farming community in the shortest possible time. These
include
1. Farmers Portal
2. Kisan Call Center
3. Fasal Bima Portal
4. Bol Kisan

Select the correct answer using the codes below.


A. 1 and 3 only
B. 1, 2 and 3 only
C. 2 and 4 only
D. 1, 2, 3 and 4 only

www.insightsias.com 22
© Insights Active Learning | All rights reserved - 78168. You may not reproduce, distribute or exploit the contents in any form without
written permission by copyright owner. Copyright infringers may face civil and criminal liability
Total Marks : 200
TEST - 10(MOCK TEST - 8) Mark Scored : 0

Your Answer :
Correct Answer : B

Answer Justification :

Justification: Others are Rice Export App, Pusa Krishi App, Agro Collect- Krishi Gyan App, e-NAM
Portal, etc.

Statement 1: Once in the Farmers’ Portal, a farmer will be able to get all relevant information on
specific subjects around his village/block /district or state.

This information will be delivered in the form of text, SMS, email and audio/video in the language
he or she understands.

Farmers will also be able to ask specific queries as well as give valuable feedback through the
Feedback module specially developed for the purpose.
7
Q Source: SS (Release ID :171225)
2 27
0 3
5
41
36 Consider the following statements.
Assertion (A): Tropical cyclones are not formed at the equator. -9
m
Reason (R): Coriolis force is absent at the equator. .coil
m a
g
7@
In the context of the above, which of these is correct?
A. A is correct, and R is an appropriate
s r aexplanation of A.
m i
B. A is correct, but R is not an appropriate explanation of A.
i t
ulk
C. A is correct, but R is incorrect.
p
D. Both A and R are incorrect.
-
sr a
Your Answer : Mi
k it : A
Correct Answer
l
Pu
Answer Justification :

Justification: Coriolis force is absent at the equator. Due to this, winds blowing from high pressure
sub-tropics to low pressure tropics do not form a circulatory pattern around a low-pressure zone.

Instead, they simply fill the low-pressure zone. This is the reason cyclones start forming only after
some distance away from the equator.

Q Source: Page 91: Chapter 10: Fundamentals of Physical Geography: 11th NCERT

37 In India, the day temperature is maximum in May and not after the summer solstice. This is due to
1. Indian Ocean Dipole and La Nino
2. Southwest monsoons occurring after Summer Solstice
3. Passing of Western cyclonic disturbances

Select the correct answer using the codes below.

www.insightsias.com 23
© Insights Active Learning | All rights reserved - 78168. You may not reproduce, distribute or exploit the contents in any form without
written permission by copyright owner. Copyright infringers may face civil and criminal liability
Total Marks : 200
TEST - 10(MOCK TEST - 8) Mark Scored : 0

A. 2 only
B. 2 and 3 only
C. 3 only
D. 1 only

Your Answer :
Correct Answer : A

Answer Justification :

Justification: Statement 1: The Indian Ocean Dipole (IOD), also known as the Indian Nino, is an
irregular oscillation of sea-surface temperatures. The IOD also affects the strength of monsoons
over the Indian subcontinent. But, this is not the causal factor.

Statement 2: The southwest monsoons arrive at the Kerala coast and advance further inland in June
and July. These winds have cooling effect and thus summer solstice in June is cooler than May.

Statement 3: Western disturbances largely affect India in the winter season. 22


77
5 03
Q Source: Page 86: Fundamentals of Physical Geography: 11th NCERT
9 41
-
m
38 Consider the following statements. i l .co
Assertion (A): Tribal made products cannot be sold on m ae-commerce websites in India.
Reason (R): Only TRIFED is authorized to sell tribal
7 @g products under the “TRIBES INDIA” showroom
sra
brand.
i
k i tofmthese is correct?
ulappropriate explanation of A.
In the context of the above, which
A. A is correct, and R ispan
a -
r
B. A is correct, but R is not an appropriate explanation of A.
is but R is correct.
M
C. A is incorrect,
D. BothkAitand R are incorrect.
l
Pu
Your Answer :
Correct Answer : D

Answer Justification :

Justification: In order to have institutional support to marketing initiatives, TRIFED has formally
signed an MOU with M/s Amazon Seller Services Pvt. Ltd for sale of tribal products through leading
e-commerce giant www.amazon.in at a mutually agreed terms & conditions.

“TRIBES INDIA” showrooms managed by TRIFED, offer a wide range of Tribal Products from
different parts of the country, which include Metal Craft, Tribal Textiles. These can be outsourced
to amazon for sell.

Q Source: http://pib.nic.in/newsite/PrintRelease.aspx?relid=171082

39 Consider the following statements about Lieutenant-governors.

www.insightsias.com 24
© Insights Active Learning | All rights reserved - 78168. You may not reproduce, distribute or exploit the contents in any form without
written permission by copyright owner. Copyright infringers may face civil and criminal liability
Total Marks : 200
TEST - 10(MOCK TEST - 8) Mark Scored : 0

1. They are appointed by the Chief Minister.


2. They hold office for a term of 5 years.
3. There is no provision of impeachment for the Lieutenant-governor.

Select the correct answer using the codes below.


A. 1 and 3 only
B. 2 and 3 only
C. 1 and 2 only
D. 1, 2 and 3 only

Your Answer :
Correct Answer : B

Answer Justification :

Justification: Provisions for Governor and Lieutenant Governor are similar.


77
22 by: Dismissal by
The term of governor’s office is normally 5 years but it can be terminated earlier
3
50pleasure the governor
the president on the advice of the prime minister of the country, at whose
1
holds office or Resignation by the governor. 94
-
m
There is no provision of impeachment, as it happens for the
i l .copresident.
m a
@g
Same applies for the LG.
a 7
Q Source: Appointment of A&N Islands isrLG
k i tm
p
40 Factors that affect temperature ulat a particular region include
a -
r
s and local winds
1. Circulation of planetary
i
M of the place
2. Altitude and terrain
t
i
kthe region from poles or equator
3. Distance lof
u
P of Ocean waves
4. Movement

Select the correct answer using the codes below.


A. 2 and 3 only
B. 1, 2 and 3 only
C. 1 and 4 only
D. 1, 2, 3 and 4

Your Answer :
Correct Answer : B

Answer Justification :

Justification: Ocean Waves do not induce movement of water in the ocean from one place to
another. Movement of water is done by ocean currents.

If water is not moved from one place to the other, there is no circulation of temperature, nutrients,

www.insightsias.com 25
© Insights Active Learning | All rights reserved - 78168. You may not reproduce, distribute or exploit the contents in any form without
written permission by copyright owner. Copyright infringers may face civil and criminal liability
Total Marks : 200
TEST - 10(MOCK TEST - 8) Mark Scored : 0

water density or salinity.

So, ocean waves affect the temperature distribution the least. Ocean ‘currents’ do. Ocean currents,
warm or cold, carry heat/coolness from one sea region to the other and affect the temperature
distribution.

Q Source: Page 82: Fundamentals of Physical Geography: 11th NCERT

41 The Mouvement des entreprises de France (MEDEF), recently seen in news is

A. the largest employer federation in France


B. a non-governmental organization for unorganized sector workers
C. a MNC that specializes in wildlife products and services
D. None of the above

Your Answer : 7
Correct Answer : A 2 27
0 3
5
Answer Justification : 41
-9
Background: Mr. Pierre Gattaz, President of MEDEF called
. com
on Prime Minister Narendra Modi
recently. i l a
g m
@India’s
The Prime Minister noted that France is one7of Top 10 sources of Foreign Direct Investment,
s r a
and invited French companies to take advantage of the improving business climate and growing
investment opportunities in India.tm
i
u lki
Learning: Also known as- p "Movement of the Enterprises of France", it has more than 750,000
the
sra of them being small and medium enterprises (SMEs) with fewer than 50
member firms, 90 percent
i
employees.
k itM
l
MEDEFPuis engaged in lobbying at local, regional, national, and EU-wide levels.
Every year, MEDEF International organises a number of delegations of French business leaders
with tangible projects to targeted countries, especially developing countries.

Q Source: AKT/NT (Release ID :171137)

42 In percentage terms, water vapour in the atmosphere, generally


1. Increases as we move to higher grounds
2. Decreases towards the poles if we start from equator

Which of the above is/are correct?


A. 1 only
B. 2 only
C. Both 1 and 2
D. None

www.insightsias.com 26
© Insights Active Learning | All rights reserved - 78168. You may not reproduce, distribute or exploit the contents in any form without
written permission by copyright owner. Copyright infringers may face civil and criminal liability
Total Marks : 200
TEST - 10(MOCK TEST - 8) Mark Scored : 0

Your Answer :
Correct Answer : B

Answer Justification :

Justification: Water vapour depends on both temperature and precipitation.

In the warm and wet tropics, it may account for four per cent of the air by volume, while in the dry
and cold areas of desert and polar regions, it may be less than one per cent of the air.

Since polar regions have a cold climate and poor precipitation, the capacity of the air as well as the
moisture available as lesser, leading to a lower water vapour content.

Q Source: Chapter 8: Fundamentals of Physical Geography: 11th NCERT

43 The movement of Sun from aphelion to Perihelion does not bring a lot of variation in the solar output
to the earth, despite earth being closer to the Sun on a perihelion, due to which of 7 7 factors?
these
2 2
1. Land and sea breezes
5 03
2. Discharge of particles due to solar wind 1
3. Milankovitch effect 94 -
. com
Select the correct answer using the codes below.
a i l
A. 2 and 3 only m
B. 1 and 2 only
7 @g
C. 1 only
isra
D. 1, 2 and 3 tm i
lk
Your Answer : - pu
Correct Answer : Csra
M i
l k it
Answer Justification :
P u
Justification: At perihelion, earth is the nearest to the sun receiving more solar insolation than at
aphelion.

Statement 2: The solar wind is a stream of charged particles released from the upper atmosphere of
the Sun. Planets with a weak or non-existent magnetosphere is subject to atmospheric stripping by
the solar wind. It is not related with the distribution of weather.

Statement 1: Distribution of land and sea, heat transfer by way of sea and land breezes etc
redistribute the increased insolation so that the effects are not felt largely.

Statement 3: Milankovitch effect refer to change in axis, shape and tilt of earth’s spin. It is relevant
to long term climate change, and not in this context which is about short-term weather changes.

Q Source: Page 79: Fundamentals of Physical Geography: 11th NCERT

44 Consider the following about the Virgo detector.

www.insightsias.com 27
© Insights Active Learning | All rights reserved - 78168. You may not reproduce, distribute or exploit the contents in any form without
written permission by copyright owner. Copyright infringers may face civil and criminal liability
Total Marks : 200
TEST - 10(MOCK TEST - 8) Mark Scored : 0

1. It is an instrument located on the highest mountain of Antarctica.


2. It tracks gravitational waves.
3. It mainly relies on electromagnetic signals such as light strokes in the sky.

Select the correct answer using the codes below.


A. 1 and 2 only
B. 2 only
C. 2 and 3 only
D. 1 and 3 only

Your Answer :
Correct Answer : B

Answer Justification :

Justification: Statement 1 and 2: The Virgo detector is an underground L-shaped instrument that
tracks gravitational waves using the physics of laser light and space. 77
3 22
The Virgo collaboration includes more than 280 physicists and engineers
1 50belonging to 20 different
4
-9
European research groups.

com
Statement 3: Known as interferometers, these high-tech underground
. stations do not rely on light in
i l
a in space and can pick up the “chirp”
the sky like a telescope does, but instead sense vibrations
m
created by a gravitational wave.
7 @g
Q Source: isra
i tm
http://www.nature.com/news/european-detector-spots-its-first-gravitational-wave-1.22690
k
p ul
a -
sr
45 It is the western boundary
i current of the southwest Indian Ocean. It acts as an oceanic convergence
zone. This ocean M current is
it
A. P ulk
Labrador
B. Gulf current
C. Benguela
D. Agulhas

Your Answer :
Correct Answer : D

Answer Justification :

Learning: It flows down the east coast of Africa from 27°S to 40°S. It is narrow, swift and strong. It
is even suggested that the Agulhas is the largest western boundary current in the world ocean.

The Agulhas acts as an oceanic convergence zone. Due to mass continuity this drives surface
waters down, resulting in the upwelling of cold, nutrient rich water south of the current.

www.insightsias.com 28
© Insights Active Learning | All rights reserved - 78168. You may not reproduce, distribute or exploit the contents in any form without
written permission by copyright owner. Copyright infringers may face civil and criminal liability
Total Marks : 200
TEST - 10(MOCK TEST - 8) Mark Scored : 0

Additionally, the convergence tends to increase the concentration of plankton in and around
the Agulhas. Both of these factors result in the area being one of enhanced primary
productivity as compared to the surrounding waters.

Q Source: Page 83: Fundamentals of Physical Geography: 11th NCERT

46 The Prime Minister Shri Narendra Modi has constituted the Economic Advisory Council to the Prime
Minister (EAC-PM). It consists of
1. Minister of Finance
2. Chairman, NITI Aayog
3. Secretary, Department of Economic Affairs
4. Deputy Governor, RBI

Select the correct answer using the codes below.


A. 1 and 2 only
7
B. 2, 3 and 4 only
2 27
C. 2 only 0 3
5
D. 1, 3 and 4 only 41
-9
Your Answer :
. com
Correct Answer : C ai l
gm
Answer Justification : 7 @
isra
t
Justification: The EAC-PM is aniindependentm body to give advice on economic and related issues to
ul k
the Government of India, specifically to the Prime Minister.
- p
a
the EAC-PM is M
isr consists of economists of high repute and eminence. The composition of
The five-member council
as follows:
l k it
Pu
Dr. Bibek Debroy, Member, NITI Aayog - Chairman

Dr. Surjit Bhalla- Part-time Member

Dr. Rathin Roy- Part-time Member

Dr. Ashima Goyal- Part-time Member

Shri Ratan Watal, Principal Advisor, NITI Aayog – Member-Secretary

It will address issues of macroeconomic importance and presenting views thereon to the Prime
Minister. This could be either suo-motu or on reference from the Prime Minister or anyone else.

www.insightsias.com 29
© Insights Active Learning | All rights reserved - 78168. You may not reproduce, distribute or exploit the contents in any form without
written permission by copyright owner. Copyright infringers may face civil and criminal liability
Total Marks : 200
TEST - 10(MOCK TEST - 8) Mark Scored : 0

Q Source: AKT/HS (Release ID :171108)

47 The International Cospas-Sarsat Programme concerns with

A. Displaying coordinates to trans-national flights in events of bad and unpredictable weather


B. Generating an Earth mapping system to monitor climate change
C. Providing geospatial information services to commercial and non-state actors
D. Detecting and locating radio beacons activated by persons, aircraft or vessels in distress

Your Answer :
Correct Answer : D

Answer Justification :

Learning: It is a treaty-based, non-profit, intergovernmental, humanitarian cooperative of 43


nations and agencies dedicated to detecting and locating radio beacons activated by
7 7 persons,
2
32 that can take
aircraft or vessels in distress, and forwarding this alert information to authorities
0
action for rescue. 5
9 41
The system utilizes a network of satellites that provide coverage-anywhere on Earth.
. com
l
ai to over 200 countries and territories at no
m
Distress alerts are detected, located and forwarded
g
cost to beacon owners or the receiving 7 @
government agencies.
isra
k i tm
l System also incorporate transponder(s) for receiving distress
The Indian National Satellite
u
p and rescue missions in the South Asian and Indian Ocean Region, as
alert signals for search
-
sra of the Cospas-Sarsat program.
ISRO is a member
i
k itM
ul
PCospas-Sarsat was conceived and initiated by Canada, France, the United States, and the
former Soviet Union in 1979.

Q Source: Surprise questions

48 Rainfall generally increases as we proceed from

A. Lower to higher altitude


B. Coastal areas to interior areas
C. Poles to equator
D. All of the above

Your Answer :
Correct Answer : C

Answer Justification :

www.insightsias.com 30
© Insights Active Learning | All rights reserved - 78168. You may not reproduce, distribute or exploit the contents in any form without
written permission by copyright owner. Copyright infringers may face civil and criminal liability
Total Marks : 200
TEST - 10(MOCK TEST - 8) Mark Scored : 0

Justification: Option A: Nothing concrete can be said about the pattern here. It depends on the
latitude and surrounding terrain. For e.g. for similar altitudes, ladakh and NE states would witness
different precipitation.

Option B: Coastal areas higher evaporation, moisture and cloud formation than interior areas.
Moreover, any moisture-laden winds from ocean will first hit the coastal areas and then interior
areas. Moisture would go on reducing as the wind advance further in. So, B is incorrect.

Option C: Poles receive lesser solar insolation, have lesser evaporation and rainfall. Reverse is true
for equator.

Q Source: Page 101: Fundamentals of Physical Geography: 11th NCERT

49 Consider the following statements.


Assertion (A): Hyperinflation can lead to a loss of confidence in the currency.
Reason (R): Inflation erodes the value of currency.
7
2 27
In the context of the above, which of these is correct?
0 3
5
A. A is correct, and R is an appropriate explanation of A.
41
B. A is correct, but R is not an appropriate explanation of A. -9
C. A is correct, but R is incorrect.
. com
D. Both A and R are incorrect. i l a
m
Your Answer :
7 @g
Correct Answer : A
isra
i t m
Answer Justification :
p ulk
a -
sr if inflation in India is 5000%, then the very next year the value of Rs. 5000
Justification: For e.g.
i
will be near Rs.M
100. It also leads to depreciation in the exchange rate of the currency.
l k it
Pu people lose confidence in the currency and holding it becomes a risky proposition. So,
As a result,
people switch to other forms of wealth like Gold, Foreign Currency (also known as “inflation proof”
assets) etc.

It is one of the important reasons for the recent Gold rush in India. Introducing inflation-indexed
bonds, Gold bonds is one way of reducing the demand of unproductive physical assets.

Q Source: Chapter 7: Indian Economy – Ramesh Singh

50 A greenhouse is made up of glass. What is the property of this glass?

A. It is transparent to incoming short wave solar radiation and opaque to outgoing long wave
radiation.
B. It is opaque to incoming short wave solar radiation and transparent to outgoing long wave
radiation.
C. It is transparent to incoming long wave solar radiation and opaque to outgoing short wave
radiation.

www.insightsias.com 31
© Insights Active Learning | All rights reserved - 78168. You may not reproduce, distribute or exploit the contents in any form without
written permission by copyright owner. Copyright infringers may face civil and criminal liability
Total Marks : 200
TEST - 10(MOCK TEST - 8) Mark Scored : 0

D. It is opaque to incoming long wave solar radiation and transparent to outgoing short wave
radiation.

Your Answer :
Correct Answer : A

Answer Justification :

Learning: The glass allows in more radiation and prevents the long wave radiation going outside
the glass house, causing the temperature inside the glasshouse structure warmer than outside.

When you enter a car or a bus, during summers, where windows are closed, you feel more heat than
outside.

Likewise during winter the vehicles with closed doors and windows remain warmer than the
temperature outside.
7
Q Source: Page 108: Fundamentals of Physical Geography: 11th NCERT
2 27
0 3
5
51 The National Wildlife Action Plan (NWAP) for 2017-31 adopts a “landscape9 41
approach” to conserve
-
om wildlife occurs
wildlife. What does this approach mean?
i
1. To conserve all the wildlife rather than the areas where l . c
specific
m
2. Focussing on conservation of wildlife by establishingasmaller National parks and Sanctuaries
3. Doing away with the system of Reserved Forests @ g
s r a7
Select the correct answer using them
i
codes below.
t
A. 1 only
u lki
B. 2 and 3 only - p
C. 1 and 2 only sra
D. 1, 2 and 3M
i
l k it
Your P u :
Answer
Correct Answer : A

Answer Justification :

Justification: The plan adopts a “landscape approach” in conservation of all wildlife – uncultivated
flora and fauna – that have an ecological value to the ecosystem and to mankind irrespective of
where they occur. It gives special emphasis to recovery of threatened species of wildlife while
conserving their habitats.

This is a key strategic change in the plan.

This means that while till now programmes and plans related to wildlife were focused on and
around national parks and sanctuaries, now the strategies would be based on the landscape of the
region that may not be limited to a reserve forest system alone.

Q Source: As mentioned above

www.insightsias.com 32
© Insights Active Learning | All rights reserved - 78168. You may not reproduce, distribute or exploit the contents in any form without
written permission by copyright owner. Copyright infringers may face civil and criminal liability
Total Marks : 200
TEST - 10(MOCK TEST - 8) Mark Scored : 0

52 Which of these constitutes the highest weightage in Consumer Price Index (Combined)?

A. Housing
B. Clothing and associated items
C. Fuel and light
D. Food and associated items

Your Answer :
Correct Answer : D

Answer Justification :

Learning: CPI is disaggregated at the rural and urban levels. The new overall all India CPI is a
weighted average of the two.

Picture below: Left column CPI Rural, Middle Column CPI Urban, and Right column CPI Combined
weights of various categories. 77 2
2
503
1
- 94
. com
a i l
m
7 @g
isra
it m
l k
u http://www.onemint.com/2013/02/25/what-is-indias-cpi/
You can read more about itphere
-
Q Source: Chapteri raIndian Economy – Ramesh Singh
s7:
k itM
l
Pu rhythm in human beings affects
53 The circadian
1. Sleep patterns
2. Blood Pressure
3. Hormone Release
4. Body Temperature

Select the correct answer using the codes below.


A. 1 and 2 only
B. 2, 3 and 4 only
C. 1 and 3 only
D. 1, 2, 3 and 4

Your Answer :
Correct Answer : D

Answer Justification :

www.insightsias.com 33
© Insights Active Learning | All rights reserved - 78168. You may not reproduce, distribute or exploit the contents in any form without
written permission by copyright owner. Copyright infringers may face civil and criminal liability
Total Marks : 200
TEST - 10(MOCK TEST - 8) Mark Scored : 0

Background: Three US scientists have won the Nobel Prize in Physiology or Medicine “for their
discoveries of molecular mechanisms controlling the circadian rhythm,” otherwise known as our
biological clock.

Jeffrey C. Hall, Michael Rosbash and Michael W. Young were awarded the prize for their research
on how plants, humans and animals adapt their biological rhythm to synchronize with our planet’s
day and night cycle, as the earth rotates, in order to control their daily life.

Justification: All living organisms on Earth have an internal clock, known as the circadian rhythm,
which in humans underlies why we are awake during the day and sleep at night.

But our biological clock also helps regulate eating habits, hormone release, blood pressure and
body temperature.

A person’s well-being is affected when there is a “temporary mismatch between our external
environment and this internal biological clock.”
7
2 27
3
1 50a number of time zones
For example, disruption to our clocks when someone travels across
results in jet lag. 94 -
. com
i l
a lead to increased risk for a number of
An imbalance between lifestyle and rhythm could
m
@g as diabetes and cancer, and neurodegenerative
diseases including metabolic diseases, such
7
a
diseases, such as Alzheimer’s disease.
sr
m i
it
Learning: Research on the body
p ulk clock has helped scientists improve health. Many drugs now on
the market work best when
a - taken at the right time. The cholesterol-cutting drug Mevacor, for
s
example, is taken iat r
night because levels of the enzyme it targets are highest then. The same is true
t M used to reduce blood pressure.
for low-dose aspirin
ulki
P
Q Source:
http://www.insightsonindia.com/2017/10/03/insights-daily-current-affairs-03-october-2017/

54 It was on the list of top 30 nominees for the "New Seven Wonders of the World". It is dedicated to a
form of Parvati, and her consort. Historical evidences of the temple are found dating from early AD
century. Located in Southern India, it is

A. Rajarajeshwara Temple
B. Meenakshi Temple
C. Lepakshi Temple
D. Kailash Temple

Your Answer :
Correct Answer : B

Answer Justification :

www.insightsias.com 34
© Insights Active Learning | All rights reserved - 78168. You may not reproduce, distribute or exploit the contents in any form without
written permission by copyright owner. Copyright infringers may face civil and criminal liability
Total Marks : 200
TEST - 10(MOCK TEST - 8) Mark Scored : 0

Learning: Meenakshi Temple is a historic Hindu temple located on the southern bank of the Vaigai
River in the temple city of Madurai, Tamil Nadu, India. It is dedicated to Meenakshi, a form of
Parvati, and her consort, Sundareswar, a form of Shiva. The temple forms the heart and lifeline of
the 2,500-year-old city of Madurai.

Sri Meenakshi Sundareswarar Temple in Madurai has been adjudged the cleanest iconic place in
the country under Swachhta Hi Seva (cleanliness is service) programme. The temple earned the top
slot from among 10 iconic places selected under the Swachh Bharath Mission in the country.

Q Source:
http://www.thehindu.com/news/national/tamil-nadu/how-meenakshi-temple-attained-the-cleanliness-
feat/article19789478.ece

55 Which of the following characterize Boreal forests.


1. It occurs at high altitudes.
2. It contains a thin cover of acidic soil that is poor in nutrients. 7
27
3. It contains coniferous forests of regions such as North America and Northern Europe.
2
3
1 50
Select the correct answer using the codes below. 4
A. 1 only -9
B. 2 and 3 only
. com
C. 1 and 2 only ai l
D. 1, 2 and 3 g m
a7@
Your Answer :
m isr
Correct Answer : D
l k it
p u
Answer Justificationa: -
i sr
Justification: M
t The Boreal Forest is the term applied to the huge area of dense coniferous forests of
l k i
Pu
North America, northern Europe and Asia occurring at high latitudes where the climate is
characterised by very cold winters, usually about 40cm of rainfall and a short summer growing
season.

The dominant species are evergreens, such as pine, spruce, fir, whose water loss through
transpiration is low in winter at the time when the soils may be permanently frozen.

Learning: Unlike tropical forests which have a huge range of species, the boreal forest usually
have rather few species of trees, shrubs and plants.

The species present are able to get their nutrient requirements from the very shallow layer of soil
above the permafrost.

The trees present have a notable climatic tolerance and survive on rather infertile soils.

Q Source: Page 130: Fundamentals of Physical Geography: 11th NCERT

56 Consider the following statements.

www.insightsias.com 35
© Insights Active Learning | All rights reserved - 78168. You may not reproduce, distribute or exploit the contents in any form without
written permission by copyright owner. Copyright infringers may face civil and criminal liability
Total Marks : 200
TEST - 10(MOCK TEST - 8) Mark Scored : 0

1. They are consecutive series of alluvial fans forming along the edge of a linear mountain range.
2. They usually occur in arid climates.

The above refer to?


A. Bajada
B. Loess
C. Sial
D. Ex-bow fans

Your Answer :
Correct Answer : A

Answer Justification :

Learning: A bajada consists of a series of coalescing alluvial fans along a mountain front. These
fan-shaped deposits form from the deposition of sediment within a stream onto flat land at the base
of a mountain 77
3 22
Surface of this feature undulates in a rolling fashion as one moves from5 0 centre of one alluvial
the
4 1
-9
fan to another.

. com
ai l
m
7 @g
isra
k i tm
p ul
a -
isr
M
l k it
Pu

Q Source: Glossary: Fundamentals of Physical Geography: 11th NCERT

57 CRISIL, often seen in news, is a

A. An International NGO focussed on disaster management


B. Credit Rating agency headquartered in India
C. A global Energy finance company
D. Group of LDCs committed to development

www.insightsias.com 36
© Insights Active Learning | All rights reserved - 78168. You may not reproduce, distribute or exploit the contents in any form without
written permission by copyright owner. Copyright infringers may face civil and criminal liability
Total Marks : 200
TEST - 10(MOCK TEST - 8) Mark Scored : 0

Your Answer :
Correct Answer : B

Answer Justification :

Justification: CRISIL (formerly Credit Rating Information Services of India Limited) is a global
analytical company providing ratings, research, and risk and policy advisory services.

CRISIL’s majority shareholder is Standard & Poor's, a division of McGraw Hill Financial and
provider of financial market intelligence

Learning: It assesses several aspects of Indian economy.

For e.g. as per its recent estimates, if all affected states also announce farm loan waivers the Uttar
Pradesh, Maharashtra, Karnataka and Punjab did, then the total cost the exchequer could be upto
Rs 2.5 lakh crore or 0.5% of GDP.
7
Q Source: Frequently in news
2 27
3
1 50
4 month in their national
9next
58 For the first time, Nepal and India will undertake a joint tiger census
-
parks, forests and protected areas adjoining the two countries using a globally-recognised method.
What is this method? . com
ai l
m
@g in tiger habitats as well as buffer zones
A. Recording movements of tigers using cameras
7
B. Estimating tiger population based r
s a the consumption of local feed including animals, reptiles
on
and birds by tigers m i
k
C. Demarcating zones that lare itdevoid of movement of any animals that could potentially be tiger
zones p u
-
sra
D. None of the above
i
Your Answer k it: M
l
PuAnswer : A
Correct

Answer Justification :

Learning: The last tiger count conducted by Nepal in 2013 puts the number of adult tigers around
200 in the Himalayan country.

The last tiger count conducted by Nepal in 2013 puts the number of adult tigers around 200.

At the International Tiger Conference in Russia in 2010, participating countries including Nepal had
made a commitment to double the tiger population by 2022. This means Nepal would have at least
250 tigers, 100% increase from its 2010 tiger count which had put the number of the big cat at 125.

Q Source:
http://www.insightsonindia.com/2017/10/03/insights-daily-current-affairs-03-october-2017/

59 Evidence of multiple advances and retreats of glaciers, and the sediment deposits in glacial lakes reveal

www.insightsias.com 37
© Insights Active Learning | All rights reserved - 78168. You may not reproduce, distribute or exploit the contents in any form without
written permission by copyright owner. Copyright infringers may face civil and criminal liability
Total Marks : 200
TEST - 10(MOCK TEST - 8) Mark Scored : 0

1. The occurrence of warm and cold periods in the history of earth


2. Repeated attacks of permafrost on the glacial moraines over centuries

Which of the above is/are correct?


A. 1 only
B. 2 only
C. Both 1 and 2
D. None

Your Answer :
Correct Answer : A

Answer Justification :

Justification: Statement 1: An interglacial period (or alternatively interglacial) is a geological


interval of warmer global average temperature lasting thousands of years that separates
consecutive glacial periods within an ice age. 77 2
2
It is a strong evidence of climate change. 5 03
1
- 94
Statement 2: Moraines are glacial deposits (soil). Permafrost is frozen carbon within rocks. It does
o
not attack or erode moraines. Permafrost is a deposit thatcis
m
revealed only when the rocks melt and
.
are exposed. ail m
@ g
Learning: Tree rings also show evidence for7 wet and dry periods. Also, for e.g. near Rajasthan,
sra with 2000-1700 BC supporting Harappan civilization.
3000-1700 BC was a period of higher irainfall,
tm
Dry conditions accentuated sinceithen.
p ulk
These are used to study -phenomenon of climate change in a region.
i sra
Q Source: Page
t M107: Fundamentals of Physical Geography: 11th NCERT
lk i
P u
60 Consider the following statements.
1. The gravitation force (g) is not the same at different latitudes on the surface of the earth.
2. Uneven distribution of mass of material within the earth influences the gravitational force as
experienced within the earth.

Which of the above is/are correct?


A. 1 only
B. 2 only
C. Both 1 and 2
D. None

Your Answer :
Correct Answer : C

Answer Justification :

www.insightsias.com 38
© Insights Active Learning | All rights reserved - 78168. You may not reproduce, distribute or exploit the contents in any form without
written permission by copyright owner. Copyright infringers may face civil and criminal liability
Total Marks : 200
TEST - 10(MOCK TEST - 8) Mark Scored : 0

Justification: Statement 1: It is greater near the poles and less at the equator. This is because of
the distance from the centre at the equator being greater than that at the poles.

Statement 2: Distribution of mass affects the effective mass that is applying the gravitational force.
Since not all earthly mass has same density, there is bound to be a difference in the way
gravitational force is experienced within the earth.

Q Source: https://phys.org/news/2016-11-theory-gravity-dark.html

61 Moderate and Milder climate in the Southern Hemisphere as compared to the Northern hemisphere,
for the same latitude, can be explained by
1. Continentality
2. Photoperiodism

Which of the above is/are correct?


A. 1 only 7
B. 2 only
2 27
C. Both 1 and 2 0 3
5
D. None 41
-9
Your Answer :
. com
Correct Answer : A ai l
gm
Answer Justification : 7 @
isra
m
t Southern Hemisphere has significantly more ocean and much
Justification: This is so becauseithe
k
l down more slowly than land. The movement of sea and land
puclimate.
less land; water heats up and cools
breeze helps to moderate- the
i sra
M where in Northern hemisphere there is more land, temperatures reach both
On the other hand,
t
extremes,lk i
for e.g. in Central Asia or Siberia.
P u
Statement 2: Photoperiodism is related to plants reaction to light, it isn’t relevant here.

Q Source: Page 83: Fundamentals of Physical Geography: 11th NCERT

62 The project SECURE Himalaya has been launched by

A. Union Environment ministry in association with the United Nations Development Programme
(UNDP)
B. World Wildlife Fund (WWF) in collaboration with UNESCO
C. International Union for Conservation of Nature (IUCN)
D. United Nations Environment Programme (UNEP) jointly with IUCN

Your Answer :
Correct Answer : A

www.insightsias.com 39
© Insights Active Learning | All rights reserved - 78168. You may not reproduce, distribute or exploit the contents in any form without
written permission by copyright owner. Copyright infringers may face civil and criminal liability
Total Marks : 200
TEST - 10(MOCK TEST - 8) Mark Scored : 0

Answer Justification :

Learning: The Centre has launched a six-year project- called SECURE Himalaya– to ensure
conservation of locally and globally significant biodiversity, land and forest resources in the high
Himalayan ecosystem spread over four states in India.

The SECURE – securing livelihoods, conservation, sustainable use and restoration of high
range Himalayan ecosystems – is meant for specific landscapes.

It includes Changthang (Jammu and Kasmir), Lahaul – Pangi and Kinnaur (Himachal Pradesh),
Gangotri – Govind and Darma – Byans Valley in Pithoragarh (Uttarakhand) and Kanchenjunga
– Upper Teesta Valley (Sikkim).

Benefits: Protection of snow leopard and other endangered species and their habitats is one of the
7
key components of the project which will also focus on securing livelihoods of the7people in the
2 2
region and enhancing enforcement to reduce wildlife crime. 3
1 50
Enhanced enforcement efforts and monitoring under the project will
- 94also curb illegal trade in some
o m
medicinal and aromatic plants which are among the most threatened species in these landscapes.

Q Source: a il.c
gm
http://www.insightsonindia.com/2017/10/03/insights-daily-current-affairs-03-october-2017/
a 7@
is r
63 What is/are the difference(s) between
i t m
extrusive and intrusive rocks?
k
ul magma, whereas intrusive rocks are formed from lava.
1. Extrusive rocks are formed from
p
2. Extrusive rocks are fine -grained, whereas intrusive rocks are coarse-grained.
a a much longer duration of time compared to intrusive rocks.
srover
i
3. Extrusive rocks form

k itM
Select thel correct answer using the codes below.
u
A. P2 only
B. 1 and 3 only
C. 3 only
D. 1 only

Your Answer :
Correct Answer : A

Answer Justification :

Concept: The key difference between intrusive and extrusive rocks is that the intrusive rocks are
formed from magma whereas the extrusive rocks are formed from lava.

Rest of the differences follow from this basic structure.

Intrusive rocks: With no air to cool the magma, these rocks are formed very slowly. Composition
of these rocks reflects presence of large crystals. These crystals interlock to form the rock.

www.insightsias.com 40
© Insights Active Learning | All rights reserved - 78168. You may not reproduce, distribute or exploit the contents in any form without
written permission by copyright owner. Copyright infringers may face civil and criminal liability
Total Marks : 200
TEST - 10(MOCK TEST - 8) Mark Scored : 0

These rocks take a very large amount of time to solidify and they remain buried deep inside
the surface of the earth being surrounded by country rocks that have been there already.

Very slow cooling means that these rocks remain coarse-grained.

Some of the perfect examples of intrusive rocks are the diorite, gabbro, and granite.

Much of the core of various mountain ranges around the world is made up of these intrusive
rocks.

Extrusive Rocks: Sometimes, molten rocks find a way to come out of the surface of the earth
through cracks and openings.

7
7 contact with air.
This magma flows in the form of lava and cools down quickly as it comes 2
into
0 32
5
41
9 of the surface of the earth are
Igneous rocks that are formed from the magma that pours-out
called extrusive rocks. om
a il.c
g m they do not get sufficient time to form
7@ and boast a fine texture.
As these rocks cool down and solidify very quickly,
large crystals. Thus, they have smallr acrystals
m is
l k it
pu Glossary: Fundamentals of Physical Geography: 11th NCERT
Q Source: Additional Research:
-
i sra
M the largest population is likely to be found in
64 Among the following,
t
lk i
A. P
u
Community
B. Ecosystem
C. Biome
D. Biosphere

Your Answer :
Correct Answer : D

Answer Justification :

Justification: In the diagram below, several levels of ecosystem have been given.

www.insightsias.com 41
© Insights Active Learning | All rights reserved - 78168. You may not reproduce, distribute or exploit the contents in any form without
written permission by copyright owner. Copyright infringers may face civil and criminal liability
Total Marks : 200
TEST - 10(MOCK TEST - 8) Mark Scored : 0

A biome harbours a large number of species, and not a particular species. Some of the major biomes
of the world are: forest, grassland, desert and tundra biomes.

However, a biosphere is the largest entity that subsumes ecosystem, biomes etc. all.
7
Q Source: Page 128: Fundamentals of Physical Geography: 11th NCERT 2 27
0 3
5
41
- 9to
65 A death insurance cover in PM Jan Dhan Yojana (PMJDY) is provided

. com
A. Families Below Poverty Line (BPL) ai l
B. Unorganized sector workers g m
C. Any person who opened a bank account a 7@under the scheme, with certain conditions
D. s r
Subscribers of the Atal Pension iYojana
itm
Your Answer : u l k
- p
Correct Answer : C
sr a
M i
it
Answer Justification
lk
:

P u
Learning: Under the PMJDY, any person who opened his first bank account between August 15,
2014, and January 26, 2015, along with a RuPay card, is eligible for a life cover of Rs 30,000 on
death due to any cause.

The idea is to provide security to poor families in both urban and rural areas who cannot afford
direct insurance and are not covered under any social security scheme.

According to the government, crores of new bank accounts were opened in 2014-15 after the PM
launched the scheme.

Q Source:
http://www.insightsonindia.com/2017/10/04/insights-daily-current-affairs-04-october-2017/

66 Consider the following with reference to the Leading Asia's Private Sector Infrastructure Fund
(LEAP).
1. It was started under the 2014 Fortaleza BRICS declaration.

www.insightsias.com 42
© Insights Active Learning | All rights reserved - 78168. You may not reproduce, distribute or exploit the contents in any form without
written permission by copyright owner. Copyright infringers may face civil and criminal liability
Total Marks : 200
TEST - 10(MOCK TEST - 8) Mark Scored : 0

2. The fund provides co-financing to sovereign infrastructure projects.


3. It will finance energy efficiency and health related projects in developing countries.

Select the correct answer using the codes below.


A. 3 only
B. 1 and 2 only
C. 2 and 3 only
D. 1, 2 and 3

Your Answer :
Correct Answer : A

Answer Justification :

Justification: Statement 1: Established in 2016, it is expected to leverage and complement ADB’s


existing non-sovereign platform to fill financing gaps
7
The Japan International Cooperation Agency (JICA) supports the fund. 2 27
0 3
5
Statement 2: It funds non-sovereign projects. 41
-9
The fund will provide financing to companies and projects,
. coasmwell as to financial intermediaries
i l
(e.g., holding companies and local currency vehicles)awhere there is a link to Infrastructure (with
the exclusion of private equity funds). g m
a 7@
Eligible countries include ADB developing r
is member countries that are also eligible for official
development assistance (ODA) from it mJapan.
l k
pu of projects ranging from infrastructure to sanitation to health and
Statement 3: it includes -a variety
public transport. sra
M i
Q Source: l k it
Pu
http://economictimes.indiatimes.com/news/economy/finance/adb-500-million-funding-soon-for-privat
e-sector-infrastructure-in-asia-and-pacific/articleshow/60927545.cms

67 The convening Ministry for the Swachh Bharat Mission is

A. Ministry of Drinking Water and Sanitation


B. Ministry of Rural Development
C. Ministry of Social Justice and Empowerment
D. Ministry of Urban Development and Poverty Alleviation

Your Answer :
Correct Answer : A

Answer Justification :

Learning: Recently the same Ministry launched the “Swachhta Hi Seva campaign”.

www.insightsias.com 43
© Insights Active Learning | All rights reserved - 78168. You may not reproduce, distribute or exploit the contents in any form without
written permission by copyright owner. Copyright infringers may face civil and criminal liability
Total Marks : 200
TEST - 10(MOCK TEST - 8) Mark Scored : 0

It was a nation-wide, fortnight-long sanitation campaign launched to highlight the government’s


flagship cleanliness initiative Swachh Bharat Mission.

The objective of the campaign is to mobilise people and reinforce the “Jan Aandolan” for sanitation
to contribute to Mahatma Gandhi’s dream of a Clean India.

Q Source: As mentioned above

68 Consider the following about LIGO.


1. It is world's largest gravitational wave observatory.
2. It comprises of two enormous laser interferometers located thousands of kilometers apart.
3. LIGO cannot see electromagnetic radiation.

Select the correct answer using the codes below.


A. 1 only
B. 2 and 3 only 7
C. 1 and 3 only
2 27
D. 1, 2 and 3 0 3
5
41
Your Answer : -9
Correct Answer : D
. com
ai l
Answer Justification : gm
@
a7
sr edge physics experiment, it is unlike a round normal-
i
Justification: Statement 1 and 2: A cutting
looking observatory. It is L-shaped
k i tm
and comprised of two enormous laser interferometers located
thousands of kilometers apart.u l
- p
r
Statement 3: LIGO sexploitsathe physical properties of light and of space itself to detect and
i
understand theMorigins of gravitational waves.
l k it
Pu optical or radio telescopes, LIGO cannot see electromagnetic radiation (e.g., visible
But, unlike
light, radio waves, microwaves) nor does it have to because gravitational waves are not part of the
electromagnetic spectrum.

In fact, electromagnetic radiation from space is so unimportant to LIGO that it is completely


isolated and sheltered from the outside world.

Learning: Gravitational waves can be explained as ripples in the fabric of space-time which can
only be caused by massive astronomical events such as neutron stars or black holes orbiting each
other so that these waves would finally radiate from them.

You can read more here https://www.ligo.caltech.edu/page/what-is-ligo

www.insightsias.com 44
© Insights Active Learning | All rights reserved - 78168. You may not reproduce, distribute or exploit the contents in any form without
written permission by copyright owner. Copyright infringers may face civil and criminal liability
Total Marks : 200
TEST - 10(MOCK TEST - 8) Mark Scored : 0

7
2 27
0 3
5
41
Q Source: Nobel Prize Physics for decisive contributions to m
-9
the LIGO detector and the observation
.c o
il
of gravitational waves
m a
@ g
69 Consider the following about Dam Rehabilitation
a 7 and Improvement Project (DRIP).
is r
1. It is a World Bank initiated project in India.
2. It is coordinated by Central Water m
itCommission.
l k
u major dams in India for improving their safety and performance.
3. It will select and focus on five
- p
s r a
Select the correctianswer using the codes below.
M
A. 1 and i3tonly
B. 2u lk
only
P
C. 1 only
D. 1, 2 and 3

Your Answer :
Correct Answer : B

Answer Justification :

Justification: The Ministry of Water Resources (MoWR), Government of India, with assistance from
the World Bank, is implementing the DRIP, which would be a six-year project.

The project envisages the rehabilitation and improvement of about 250 dams in states namely,
Kerala, Madhya Pradesh, Odisha, Tamil Nadu, Karnataka, Uttarakhand and Jharkhand.

The Central Dam Safety Organisation of Central Water Commission, assisted by a Consulting firm,
is coordinating and supervising the Project implementation.

www.insightsias.com 45
© Insights Active Learning | All rights reserved - 78168. You may not reproduce, distribute or exploit the contents in any form without
written permission by copyright owner. Copyright infringers may face civil and criminal liability
Total Marks : 200
TEST - 10(MOCK TEST - 8) Mark Scored : 0

Q Source:
http://www.insightsonindia.com/2017/10/04/insights-daily-current-affairs-04-october-2017/

70 The United Nations designated the first Monday of October of every year as World Habitat Day. UN-
Habitat’s focal areas include
1. Inclusive housing and social services.
2. Affordable and sustainable transport and energy.
3. Safe and clean drinking water and sanitation.
4. Healthy air quality
5. Job creation

Select the correct answer using the codes below.


A. 1, 2 and 5 only
B. 3 and 4 only
C. 2, 3 and 5 only
D. 1, 2, 3, 4 and 5 7
2 27
0 3
Your Answer : 5
41
-9
Correct Answer : D

Answer Justification : . com


ai l
g mon a new theme to promote sustainable
Justification: Each year, World Habitat Day takes
@
a7shelter for all.
development policies that ensure adequate
sr
m i
it
2017 theme is Housing Policies: Affordable
lk
Homes
u
It also covers: -p
i sra
M
kit protection, and restoration of green urban spaces.
Promotion,
l
Pu
Improved urban planning and slum upgrading.

Better waste management.

A safe and healthy living environment for all — with particular consideration for children,
youth, women, elderly and disabled.

Q Source: As mentioned above

71 Consider the following statements with reference to the carbon budget in India.
1. The forestry sector is a net carbon absorber.
2. The energy sector is the highest carbon emitter followed by agriculture.

www.insightsias.com 46
© Insights Active Learning | All rights reserved - 78168. You may not reproduce, distribute or exploit the contents in any form without
written permission by copyright owner. Copyright infringers may face civil and criminal liability
Total Marks : 200
TEST - 10(MOCK TEST - 8) Mark Scored : 0

Which of the above is/are correct?


A. 1 only
B. 2 only
C. Both 1 and 2
D. None

Your Answer :
Correct Answer : C

Answer Justification :

Justification: Statement 1: The energy sector accounts for two-thirds the total emissions, followed
by agriculture (20 per cent), industrial processes (5.8 per cent) and waste (3.4 per cent)

Among gases, CO2 is the largest contributor to GHG emissions at 67.25%, followed by methane with
a contribution of 26.73% and nitrous oxide, 5%.
77
22 amount to
Statement 2: The forestry sector was a net absorber; carbon absorption by forests
3
222,567 million tonnes of CO2 equivalent. 50
9 41
- Geography: 11th NCERT
Q Source: Additional Research: Page 108: Fundamentals of Physical
m
i l .co
m a roam freely. These countries include
72 Tiger range countries (TRCs) are those where the tigers
1. Bangladesh 7 @g
2. Indonesia
isra
3. Malaysia
k i tm
4. Nepal
p ul
5. Russia
a -
is r
M
Select the correct answer using the codes below.
A. 1, 3lk
it
and 4 only
B. P
u
1, 2, 4 and 5 only
C. 2 and 5 only
D. 1, 2, 3, 4 and 5

Your Answer :
Correct Answer : B

Answer Justification :

Justification: The 13 tiger range countries include Bangladesh, Bhutan, Cambodia, China, India,
Indonesia, Lao PDR, Malaysia, Myanmar, Nepal, Russia, Thailand, and Vietnam.

Tiger is an endangered animal listed in the Convention on International Trade in Endangered


Species (CITES). According to the World Wildlife Fund (WWF), tigers have lost 93% of their
historical range.

Q Source:

www.insightsias.com 47
© Insights Active Learning | All rights reserved - 78168. You may not reproduce, distribute or exploit the contents in any form without
written permission by copyright owner. Copyright infringers may face civil and criminal liability
Total Marks : 200
TEST - 10(MOCK TEST - 8) Mark Scored : 0

http://www.insightsonindia.com/2017/10/03/insights-daily-current-affairs-03-october-2017/

73 Consider the following statements.


Assertion (A): In India, both the Central and State government can make developmental and welfare
plans.
Reason (R): ‘Economic and social planning’ falls under the Concurrent List in the Seventh schedule of
the Constitution.
#40166

In the context of the above, which of these is correct?


A. A is correct, and R is an appropriate explanation of A.
B. A is correct, but R is not an appropriate explanation of A.
C. A is incorrect, but R is correct.
D. Both A and R are incorrect.

Your Answer :
7
Correct Answer : A
2 27
0 3
5
Answer Justification : 41
-9
Learning: ‘Economic and social planning’ is a concurrent o m Also, while framing the ‘Union’,
subject.
i l
‘State’ and ‘Concurrent’ list, allocating subjects and other . c provisions, the Constitution vests power
in the Union to ensure co-ordinated development m
a
in essential fields of activity while preserving the
g
7@ allotted to them.
initiative and authority of the states in the spheres
isra
Q Source: Revision: Chapter 4: Indian
i t m Economy – Ramesh Singh
p ulk
a -
74 Consider the following statements.
r
is phenomena on the photosphere of the Sun.
1. Sunspots are temporary
M
2. They appearitvisibly as bright spots compared to surrounding regions.
u lkaffect the climate on earth by changing the solar output received by earth.
P
3. They may

Select the correct answer using the codes below.


A. 1 only
B. 2 and 3 only
C. 1 and 3 only
D. 1, 2 and 3

Your Answer :
Correct Answer : C

Answer Justification :

Justification: They appear as darker spots. They reduce solar output if they increase in number.

Moreover, they reflect intense magnetic activity on the Sun at those spots.

If the solar output is changed, it would affect the temperature and wind patterns on earth causing
www.insightsias.com 48
© Insights Active Learning | All rights reserved - 78168. You may not reproduce, distribute or exploit the contents in any form without
written permission by copyright owner. Copyright infringers may face civil and criminal liability
Total Marks : 200
TEST - 10(MOCK TEST - 8) Mark Scored : 0

either a short-term weather change or if persistent may also cause a long-term climate change.

Q Source: Page 107: Fundamentals of Physical Geography: 11th NCERT

75 Generally, at the same latitude the insolation is more over the continent than over the oceans,
because
1. Oceans are cloudier
2. Continents have varied relief

Which of the above is/are correct?


A. 1 only
B. 2 only
C. Both 1 and 2
D. None

Your Answer : 7
Correct Answer : A 2 27
0 3
5
Answer Justification : 41
-9
. com Similarly, in equator, evaporation is
Justification: Statement 1: This is due to greater evaporation.
l
ai reflect incoming solar insolation and send it
high, cloudiness is high and rains are frequent. Clouds
back to space. g m
a 7@
r
Statement 2: Despite the varied terrainsand relief, amount of insolation received by land isn’t
i
i
affected so significantly as to causet m
such a difference in the total insolation received by land and
seas. ul k
- p
Q Source: Page 80: sr a
Fundamentals of Physical Geography: 11th NCERT
i
k i tM
ul to the Protocol on Environmental Protection to the Antarctic Treaty, consider the
76 With reference
P
following:
1. Any activity relating to mineral resources, other than scientific research, shall be prohibited.
2. Environmental assessment for all activities, including tourism, shall be done.
3. Member states should be prepared for emergency response actions in the area.

Select the correct answer using the codes below.


A. 1 only
B. 2 and 3 only
C. 2 only
D. 1, 2 and 3

Your Answer :
Correct Answer : D

Answer Justification :

www.insightsias.com 49
© Insights Active Learning | All rights reserved - 78168. You may not reproduce, distribute or exploit the contents in any form without
written permission by copyright owner. Copyright infringers may face civil and criminal liability
Total Marks : 200
TEST - 10(MOCK TEST - 8) Mark Scored : 0

Justification: Also known as the Antarctic-Environmental Protocol, or the Madrid Protocol, it is a


part of the Antarctic Treaty System.

It provides for comprehensive protection of the Antarctic environment and dependent and
associated ecosystems.

Other provisions (apart from the ones in the Q) are:

Article 3 states that protection of the Antarctic environment as a wilderness with aesthetic
and scientific value shall be a "fundamental consideration" of activities in the area.

Article 11 creates a Committee for Environmental Protection for the continent.

Articles 18-20 arrange for arbitration of international disputes regarding Antarctica.


7
2 27
Learning: The main mineral resource known on the continent is coal. The 3
0most valuable resources
15the Ross Sea. Exploitation of
of Antarctica lie offshore, namely the oil and natural gas fields found4in
-9
all mineral resources is banned until 2048 by the Protocol on Environmental Protection to the
m
Antarctic Treaty. co
il.
m a
Q Source: Map-based questions: Antarctica g
a 7@
r
iscoliform bacteria.
77 Consider the following statements aboutm
ittoxic to human beings.
k
1. All strains of coliform bacterialare
pu
2. They are present in large- numbers in the faeces of warm-blooded animals.
3. Coliform bacteria are sr a
a commonly used bacterial indicator of sanitary quality of foods and water.
M i
Select thelk it answer using the codes below.
correct
A. P2uand 3 only
B. 1 and 2 only
C. 3 only
D. 1, 2 and 3

Your Answer :
Correct Answer : A

Answer Justification :

Justification: Statement 1: While coliforms themselves are not normally causes of serious illness,
they are easy to culture, and their presence is used to indicate that other pathogenic organisms of
faecal origin may be present.

E. coli is a member of the coliform group. Most strains of E. coli are harmless, but some can cause
serious illness in humans.

www.insightsias.com 50
© Insights Active Learning | All rights reserved - 78168. You may not reproduce, distribute or exploit the contents in any form without
written permission by copyright owner. Copyright infringers may face civil and criminal liability
Total Marks : 200
TEST - 10(MOCK TEST - 8) Mark Scored : 0

Statement 2: Coliforms can be found in the aquatic environment, in soil and on vegetation; they are
universally present in large numbers in the feces of warm-blooded animals.

Statement 3: For e.g. in Ganges water, presence of large number of E.Coli indicates large dump of
faecal matter.

Q Source: Additional Research: National Mission for Clean Ganga

78 Which of the following regions come under the Arctic Circle?


1. Oslo
2. Greenland
3. Alaska
4. Sakhalin

Select the correct answer using the codes below.


A. 1 and 4 only 7
B. 2 and 3 only 2 27
C. 1, 2 and 3 only 0 3
5
D. 2, 3 and 4 only 41
-9
Your Answer :
. com
Correct Answer : B ai l
gm
7 @
sra
Answer Justification :

m i
Justification: i t
p ulk
a -
i sr
M
l k it
Pu

www.insightsias.com 51
© Insights Active Learning | All rights reserved - 78168. You may not reproduce, distribute or exploit the contents in any form without
written permission by copyright owner. Copyright infringers may face civil and criminal liability
Total Marks : 200
TEST - 10(MOCK TEST - 8) Mark Scored : 0

7
2 27
0 3
5
41
-9
. com
ai l
gm
7 @
isra
i t m
p ulk
a -
i sr
M
l k it
Pu

Q Source: Map based questions: Arctic

www.insightsias.com 52
© Insights Active Learning | All rights reserved - 78168. You may not reproduce, distribute or exploit the contents in any form without
written permission by copyright owner. Copyright infringers may face civil and criminal liability
Total Marks : 200
TEST - 10(MOCK TEST - 8) Mark Scored : 0

79 Polymetallic nodules (manganese nodules) are rock concretions on the sea bottom formed of
concentric layers of metal hydroxides around a core. It mainly contains
1. Nickel
2. Selenium
3. Cobalt
4. Bromium

Select the correct answer using the codes below.


A. 1 and 3 only
B. 2, 3 and 4 only
C. 1, 2 and 3 only
D. 1, 2, 3 and 4

Your Answer :
Correct Answer : A

7
Answer Justification :
2 27
0 3
Justification: Mainly iron and manganese hydroxide, it also contains1 5 and copper.
nickel
- 94
Government of India is pushing mining of these nodules fromm sea beds as they are commercially
co
attractive.
a il.
Nodule mining could affect tens of thousands ofg
m
square kilometers of deep sea ecosystems.
a 7@
r
is of years and that would make such mining an
Nodule regrowth takes decades to millions
m
itpractice.
unsustainable and non-renewable
l k
- pu
sra
Q Source:
i
http://www.thehindubusinessline.com/news/national/cabinet-okays-extension-of-contract-for-rights-t
itM
o-explore-polymetallic-nodules-in-indian-ocean/article9099822.ece
k
l
Pu
80 The concept of inter-generational equity has a bearing on
1. Principles of taxation and public borrowing
2. Sustainable Development

Which of the above is/are correct?


A. 1 only
B. 2 only
C. Both 1 and 2
D. None

Your Answer :
Correct Answer : C

Answer Justification :

Justification: Statement 1: “Ricardian equivalence” says that if government borrows too much

www.insightsias.com 53
© Insights Active Learning | All rights reserved - 78168. You may not reproduce, distribute or exploit the contents in any form without
written permission by copyright owner. Copyright infringers may face civil and criminal liability
Total Marks : 200
TEST - 10(MOCK TEST - 8) Mark Scored : 0

from the public, it can repay this debt only by increasing the taxes, on either the present generation
or the coming generations. Therefore, there should be a sense of equity in public borrowing
programs that do not put a lot of burden on the coming generations.

Statement 2: This is clear as more pollution and consumption will only deplete resources for the
coming generations.

The essential idea is that we should not compromise on the needs of the future generations against
the present generation.

Q Source: Basics of Environment

81 Consider the following statements. The Ross Sea is


1. A deep bay of the Southern Ocean in Antarctica
2. Free of ice for most of the year
3. Dominated by movements of cold counter-equatorial currents
7
2 27
Select the correct answer using the codes below. 0 3
5
A. 1 only 41
B. 2 and 3 only -9
C. 1 and 2 only
. com
D. 1 and 3 only
ai l
gm
Your Answer : 7 @
Correct Answer : A isra
i t m
Answer Justification :
p ulk
a -
sr 1: To the west of the sea lies Ross Island and to the east Roosevelt Island,
Justification: Statement
i
k itM
while the southernmost part is covered by the Ross Ice Shelf, and is about 200 miles from the South
Pole. l
Pu
Statement 2 and 3: The circulation of the Ross Sea is dominated by a wind-driven ocean gyre and
the flow is strongly influenced by three submarine ridges that run from southwest to northeast.

The circumpolar deep water current is a relatively warm, salty and nutrient-rich water mass that
flows onto the continental shelf at certain locations. But, even then Ross Sea is covered with ice for
most of the year.

Q Source: Map-based questions: Antarctica

82 Why is sedimentation harmful for reservoirs?


1. It results in rockier stream beds.
2. A lot of sedimentation may cause local flooding.
3. It adversely affects the reservoir bed habitats for the aquatic life.

Select the correct answer using the codes below.

www.insightsias.com 54
© Insights Active Learning | All rights reserved - 78168. You may not reproduce, distribute or exploit the contents in any form without
written permission by copyright owner. Copyright infringers may face civil and criminal liability
Total Marks : 200
TEST - 10(MOCK TEST - 8) Mark Scored : 0

A. 1 only
B. 2 only
C. 2 and 3 only
D. 1, 2 and 3

Your Answer :
Correct Answer : D

Answer Justification :

Justification: Statement 1 and 2: All rivers contain sediments. When a river is stilled behind a dam,
the sediments it contains sink to the bottom of the reservoir. As the sediments accumulate in the
reservoir, so the dam gradually loses its ability to store water for the purposes for which it was
built.

If sedimentation continues, it increases water level and can even cause floods.
77
Q Source:
3 22
50
http://www.thehindu.com/sci-tech/energy-and-environment/climate-change-impact-sunderbans-stea
1
dily-losing-its-famed-mangroves/article19195229.ece 4
-9
m
83 Consider the following statements. Hypertension is il. co
m a
1. A Non-communicable (NCD) disease g
2. Caused by a regular protein diet
a 7@
is r
3. dangerous because contributes to hardening of the arteries

k i tm
ul the codes below.
Select the correct answer using
p
A. 1 and 2 only
B. 2 and 3 onlyisr
a-
M
C. 1 and 3tonly
k i
D. 1,u2l and 3
P
Your Answer :
Correct Answer : C

Answer Justification :

Learning: Hypertension is a case of high blood pressure.

Blood pressure is the measure of the force of blood pushing against blood vessel walls.

The heart pumps blood into the arteries (blood vessels), which carry the blood throughout the
body.

Hypertension is harmful because it makes the heart work harder to pump blood out to the
body and contributes to hardening of the arteries, or atherosclerosis, to stroke, kidney

www.insightsias.com 55
© Insights Active Learning | All rights reserved - 78168. You may not reproduce, distribute or exploit the contents in any form without
written permission by copyright owner. Copyright infringers may face civil and criminal liability
Total Marks : 200
TEST - 10(MOCK TEST - 8) Mark Scored : 0

disease, and to the development of heart failure.

The exact causes of high blood pressure are not known, but several factors and conditions
may play a role in its development, including: Smoking; Being overweight or obese; Lack of
physical activity; Too much salt in the diet etc.

Q Source: Frequently in news

84 A high Gini score for a nation means that


1. The nation is highly economically developed
2. The nation has high economic inequality

Which of the above is/are correct?


A. 1 only
7
B. 2 only
2 27
C. Both 1 and 2
0 3
5
D. None
41
-9
Your Answer :
. com
Correct Answer : B
ai l
gm
@
a7
Answer Justification :
s r
i to represent the income distribution of a nation's
m
Justification: Gini coefficient is intended
t
lki used measure of inequality.
residents, and is the most commonly
u
- p
A nation with a Gini r a of 0 has perfect income equality, and score of 100 has perfect inequality
s score
M
(i.e. all the wealthiis owned by a single person).
t
u lki
If Gini coefficient of a Nation is high, there is no guarantee that the Nation will be wealthy or poor.
P
Q Source:
http://www.livemint.com/Opinion/WggBKdjWUaGlXNrQvHWUUJ/Addressing-Indias-income-inequali
ty.html

85 Saltwater intrusion is a common problem in

A. Hilly areas
B. Coastal areas
C. Interior of Continents
D. Arid regions

Your Answer :
Correct Answer : B

Answer Justification :
www.insightsias.com 56
© Insights Active Learning | All rights reserved - 78168. You may not reproduce, distribute or exploit the contents in any form without
written permission by copyright owner. Copyright infringers may face civil and criminal liability
Total Marks : 200
TEST - 10(MOCK TEST - 8) Mark Scored : 0

Learning: It is the movement of saline water into freshwater aquifers.

The movement happens because saltwater has a higher mineral content than freshwater, it is
denser and has a higher water pressure.

As a result, saltwater can push inland beneath the freshwater, which can lead to contamination of
drinking water sources and other consequences.

Q Source: Groundwater pollution reports frequently published in newspapers

86 Consider the following statements. Grameen Bank is


1. A microfinance organization owned by the Government of Bangladesh
2. A community development bank which provides loans to the poor without demanding collateral

Which of the above is/are correct?


A. 1 only
7
B. 2 only
2 27
C. Both 1 and 2 0 3
5
D. None 41
-9
Your Answer :
. com
Correct Answer : B
ai l
gm
@
Answer Justification :
s r a7
m i
Learning: Grameen Bank (GB) has t reversed conventional banking practice by removing the need
ki a loan is made, e.g. house) and created a banking system
u
for collateral (security againstlwhich
- p
based on mutual trust, accountability, participation and creativity.
sr a
i
M Yunus, the founder of "Grameen Bank" and its Managing Director, reasoned
Professor Muhammad
it
lk resources can be made available to the poor people on terms and conditions that are
that if financial
u
P
appropriate and reasonable, "these millions of small people with their millions of small pursuits can
add up to create the biggest development wonder.”

Q Source: http://www.thehindu.com/books/turn-the-economic-ship-around/article19747532.ece

87 In Special Economic Zones (SEZs) business and trades laws differ from the rest of the country. These
relaxations are provided in order to
1. Make SEZs Export Processing Zones thereby promoting exports
2. Establish industrial townships in SEZs

Which of the above is/are correct?


A. 1 only
B. 2 only
C. Both 1 and 2
D. None

www.insightsias.com 57
© Insights Active Learning | All rights reserved - 78168. You may not reproduce, distribute or exploit the contents in any form without
written permission by copyright owner. Copyright infringers may face civil and criminal liability
Total Marks : 200
TEST - 10(MOCK TEST - 8) Mark Scored : 0

Your Answer :
Correct Answer : A

Answer Justification :

Justification: The term special economic zone (SEZ) is commonly used as a generic term to refer to
any modern economic zone.

The relaxed policies typically regard investing, taxation, trading, quotas, customs and labour
regulations. Additionally, companies may be offered tax holidays.

The aims of the zones include: increased trade, increased investment, job creation and effective
administration. To encourage businesses to set up in the zone, financially libertarian policies are
introduced.

We will cover more on SEZs in coming tests.


7
Q Source: Frequently in news
2 27
0 3
5
41
88 Which of these seas surround Arctic Ocean?
-9
1. Beaufort Sea
. com
2. East Siberian Sea
ai l
3. Kuril Sea
gm
4. Greenland Sea @
s r a7
Select the correct answer using them i below.
codes
t
A. 1 and 3 only
u lki
B. 1, 2 and 4 only - p
C. 2 and 4 only sra
D. 1, 2, 3 andM4
i
l k it
Your P u :
Answer
Correct Answer : B

Answer Justification :

Justification:

www.insightsias.com 58
© Insights Active Learning | All rights reserved - 78168. You may not reproduce, distribute or exploit the contents in any form without
written permission by copyright owner. Copyright infringers may face civil and criminal liability
Total Marks : 200
TEST - 10(MOCK TEST - 8) Mark Scored : 0

7
2 27
0 3
5
41
-9
om
Q
Source: Map based questions: Arctic
i l . c
m a
89 Consider the following statements. A ‘Captive7power plant’@g
1. Is a Privately managed power plant is
ra
2. Subsidized by government in public
k i tminterest
l
pua Nuclear power plant
3. Is generally established near
-
s r a
Select the correctianswer using the codes below.
A. 1 onlyit
M
B. 2u lk 3 only
and
P
C. 1 and 3 only
D. 2 only

Your Answer :
Correct Answer : A

Answer Justification :

Justification & Learning: A captive power plant is a facility that is dedicated to providing a
localised source of power to an energy user.

The plants may operate in grid parallel mode with the ability to export surplus power to the local
electricity distribution network.

They may also have the ability to operate in island mode; i.e. independently of the local electricity
distribution system.

www.insightsias.com 59
© Insights Active Learning | All rights reserved - 78168. You may not reproduce, distribute or exploit the contents in any form without
written permission by copyright owner. Copyright infringers may face civil and criminal liability
Total Marks : 200
TEST - 10(MOCK TEST - 8) Mark Scored : 0

Statement 3: If there is a nuclear power plant nearby, there is no need of a captive plant.

Since the announcement of private power policy of the Government of India in 1991, a number of
proposals, including a large number of proposals from foreign promoters, have been received
through Independent Power Producer (IPP) route.

Q Source: Frequently in news

90 CHA-AM Hua Hin Declaration, seen in news sometime back, concerns

A. Infrastructure financing by developed countries


B. Support to sunshine industries
C. Policy coordination between Central banks of various countries
D. Strengthening cooperation on education in ASEAN

Your Answer : 7
Correct Answer : D 2 27
0 3
5
Answer Justification : 41
-9
Learning: The declaration intends to:
. com
ai l
Promote better understanding and appreciation ofm
the ASEAN Charter through the school
g
curriculum on ASEAN and disseminating the
a 7@
ASEAN Charter which has been translated into ASEAN
national languages; s r i
m
l kit
Give greater emphasis on the principles
u of democracy, respect for human rights and peace-oriented
- p
values in the school curriculum;

Promote betterM i sra


understanding and appreciation of different cultures, customs and faiths in the
t
ki teachers through training and exchange programmes and establishment of an online
region among
u l
P on this subject;
database

Q Source: Surprise questions:


http://www.asean.org/storage/images/archive/15thsummit/Declaration-Education.pdf

91 The Bering Sea is a marginal sea of the

A. Arctic Ocean
B. North Sea
C. Pacific Ocean
D. Beaufort Sea

Your Answer :
Correct Answer : C

Answer Justification :

www.insightsias.com 60
© Insights Active Learning | All rights reserved - 78168. You may not reproduce, distribute or exploit the contents in any form without
written permission by copyright owner. Copyright infringers may face civil and criminal liability
Total Marks : 200
TEST - 10(MOCK TEST - 8) Mark Scored : 0

Learning: The Bering Sea is separated from the Gulf of Alaska by the Alaska Peninsula.

It comprises a deep water basin, which then rises through a narrow slope into the shallower water
above the continental shelves.

The Bering Sea ecosystem includes resources within the jurisdiction of the United States and
Russia, as well as international waters in the middle of the sea (known as the "Donut Hole").

The interaction between currents, sea ice, and weather makes for a vigorous and productive
ecosystem.

Q Source: Map-based questions: Arctic

92 Operation IceBridge is a 2009–2018 NASA mission that aims to


1. Monitor changes in polar ice from a satellite
2. Create physical underground networks for clutter free movement on frozen surfaces
7
2 27
Which of the above is/are correct?
0 3
5
A. 1 only
41
B. 2 only -9
C. Both 1 and 2
. com
D. None
ai l
gm
Your Answer : 7 @
Correct Answer : D
isra
i t m
Answer Justification :
p ulk
a -
srto monitor polar ice changes, but it is not being done through a satellite, but
Justification: It was
i
M aircraft.
instead a fixed-wing
l k it
u NASA has used a satellite, ICESat (Ice, Cloud, and land Elevation Satellite), for
SinceP2003
observing polar ice. ICESat was retired in 2009 due to a technical malfunction, leaving NASA
without a satellite dedicated to ice observance.

NASA therefore introduced the IceBridge program which utilizes an aircraft to make similar
measurements.

Learning: IceBridge flights began in 2009. There are tradeoffs to using an aircraft instead of a
satellite.

One drawback is that a satellite can observe a far wider area. Also, satellites take measurements
full-time, while IceBridge aircraft measurements are limited to annual campaigns that are several
weeks long.

Aircraft, however, have the advantages of being able to carry more instruments and target and
focus on scientifically interesting areas instead of just flying a fixed path.

Q Source: Map-based questions: Antarctica

www.insightsias.com 61
© Insights Active Learning | All rights reserved - 78168. You may not reproduce, distribute or exploit the contents in any form without
written permission by copyright owner. Copyright infringers may face civil and criminal liability
Total Marks : 200
TEST - 10(MOCK TEST - 8) Mark Scored : 0

https://www.nasa.gov/mission_pages/icebridge/mission/index.html

93 Oxygen is added to the atmosphere by which of the following processes?


1. Formation and breakdown of Ozone
2. Photosynthesis by plants
3. Metabolism by legume bacteria

Select the correct answer using the codes below.


A. 2 only
B. 2 and 3 only
C. 1 only
D. 1 and 3 only

Your Answer :
Correct Answer : A
7
Answer Justification : 2 27
0 3
5
Justification: Statement 1: Formation of ozone consumes oxygen.9 41
-
. com
a i l
m
7 @g
isra
i t m
p ulk
a -
i sr
M
l k it
Pu

Statement 2: If you notice, the only major process returning oxygen is photosynthesis. This is why
afforestation can tackle climate change and pollution.

Statement 3: The process of nitrogen-fixing by bacteria does not take place in the presence of
oxygen.

Q Source: 11th NCERT: Fundamentals of Physical Geography

www.insightsias.com 62
© Insights Active Learning | All rights reserved - 78168. You may not reproduce, distribute or exploit the contents in any form without
written permission by copyright owner. Copyright infringers may face civil and criminal liability
Total Marks : 200
TEST - 10(MOCK TEST - 8) Mark Scored : 0

94 Navdanya movement focusses on

A. Empowerment of women
B. Conserving culture and sustainable development
C. Establishing an earth democracy
D. All of the above

Your Answer :
Correct Answer : D

Answer Justification :

Learning: You can read the achievements of Navdanya here


http://www.navdanya.org/site/component/content/article?id=7

It is a women centred movement for Earth Democracy based on the philosophy of “The Earth as one
2 77
Family” for conserving biodiversity, conserving knowledge of the seed and its utilization, conserving
culture, conserving sustainability. 3 2
1 50
- 94
It has created awareness on the hazards of genetic engineering, defended people's knowledge from
m change.
biopiracy and food rights in the face of globalisation and climate
o
il. c
Q Source: Surprise questions
m a
g
a 7@
95 Consider the following about the Geography r
is of the Antarctica continent.
1. It is positioned symmetrically around m
it the South Pole.
l k
- pu
2. It lies largely north of the Antarctic Circle.

Which of the above i ra correct?


sis/are
A. 1 onlyit M
B. 2u lk
only
C. PBoth 1 and 2
D. None

Your Answer :
Correct Answer : D

Answer Justification :

Justification: Positioned asymmetrically around the South Pole and largely south of the Antarctic
Circle, Antarctica is the southernmost continent and is surrounded by the Southern Ocean.

Alternatively, it may be considered to be surrounded by the southern Pacific, Atlantic, and Indian
Oceans, or by the southern waters of the World Ocean.

Q Source: Map-based questions: Antarctica

www.insightsias.com 63
© Insights Active Learning | All rights reserved - 78168. You may not reproduce, distribute or exploit the contents in any form without
written permission by copyright owner. Copyright infringers may face civil and criminal liability
Total Marks : 200
TEST - 10(MOCK TEST - 8) Mark Scored : 0

96 Which of the following characterize the famous Lake Vostok?


1. It lies below the sea level.
2. It is a brackish lake.

Which of the above is/are correct?


A. 1 only
B. 2 only
C. Both 1 and 2
D. None

Your Answer :
Correct Answer : A

Answer Justification :

Justification: Statement 1: There are a number of rivers and lakes in Antarctica, the longest river
7
in7the world.
being the Onyx. The largest lake, Vostok, is one of the largest sub-glacial lakes2
0 32
Statement 2: The surface of this fresh water lake is approximately 4,000
4 15m under the surface of the
ice, which places it at approximately 500 m below sea level.
-9
m
ooff
Learning: The lake water is estimated to have been sealed
i l . c under the thick ice sheet about 15
million years ago.
m a
g
a 7@
Living micro-organisms have been
r
is found in Lake Vostok's deep ice core drillings; despite the
complete absence of sunlight
m
it received in it.
l k
- pu
This suggestsi ra presence of a deep biosphere utilizing a geothermal system of the bedrock
sthe
k it Mthe subglacial lake.
encircling
l
Pu
Due to the disconnection of Lake Vostok with the rest of the world, it is a very interesting
puzzle for evolutionary biologists.

Q Source: Map-based questions: Antarctica

97 What are the explicit provisions for environmental protection and conservation in the Directive
Principles of State Policy?
1. It provides for a Right to Clean environment.
2. It delegates the responsibility of managing local health and sanitation to the local bodies.
3. It exhorts the government not to undertake projects that involve large scale deforestation.

Select the correct answer using the codes below.


A. 1 only
B. 2 and 3 only
C. 1 and 3 only

www.insightsias.com 64
© Insights Active Learning | All rights reserved - 78168. You may not reproduce, distribute or exploit the contents in any form without
written permission by copyright owner. Copyright infringers may face civil and criminal liability
Total Marks : 200
TEST - 10(MOCK TEST - 8) Mark Scored : 0

D. None of the above

Your Answer :
Correct Answer : D

Answer Justification :

Justification: Statement 1: This is under FR.

Statement 2: This is under 11th and 12th schedule, not DPSP.

Statement 3: This is a policy of the government, but not provided for in DPSP.

There is only one provision in Article 48A that says about Protection and improvement of
environment and safeguarding of forests and wild life.

Q Source: Revision of Previous tests: 9th NCERT Democratic Polity


7
2 27
0 3
5
41
98 Parties to the Antarctica Treaty System include
1. India -9
2. Russia
. com
3. China
ai l
4. USA m
5. Australia @g
s r a7
m i
t
Select the correct answer using the codes below.
A. 2 and 4 only
u lki
B. 2, 4 and 5 only - p
C. 1, 3 and 5 only
i sra
D. 1, 2, 3, 4 M
and 5
it
u lk
Your P
Answer :
Correct Answer : B

Answer Justification :

Justification: The treaty regulates international relations with respect to Antarctica. We will cover
more on this important treaty later.

www.insightsias.com 65
© Insights Active Learning | All rights reserved - 78168. You may not reproduce, distribute or exploit the contents in any form without
written permission by copyright owner. Copyright infringers may face civil and criminal liability
Total Marks : 200
TEST - 10(MOCK TEST - 8) Mark Scored : 0

7
2 27
0 3
5
41
-9
. com
ai l
gm
7 @ Q
sra
Source: Map-based questions: Antarctica
i
m
lkit
puof these nations?
99 Barents Sea is closest to which
-
a
M
A. Finland, Sweden
isr
and Norway
it
ulk
B. Poland, Belarus and Ukraine
C. P
France and Germany
D. Morocco, Italy and Greece

Your Answer :
Correct Answer : A

Answer Justification :

Learning:

www.insightsias.com 66
© Insights Active Learning | All rights reserved - 78168. You may not reproduce, distribute or exploit the contents in any form without
written permission by copyright owner. Copyright infringers may face civil and criminal liability
Total Marks : 200
TEST - 10(MOCK TEST - 8) Mark Scored : 0

7
2 27
0 3
5
41
-9
. com
ai l
gm
@
s r a7
Q Source: Map based questions: Arctic m i
t
u lki
- p
i sra
k itM
100 In international law, a condominium

P ul
A. is a refugee territory over from which no nation has the right to expel the protectorates
B. is an economic entity that is free to enter into or exit from agreements made independently
with sovereign governments
C. is an international organization that has been granted observer status in any of the United
Nations Agencies or arms
D. is a political territory in or over which multiple sovereign powers formally agree to share
equal dominium or rights

Your Answer :
Correct Answer : D

Answer Justification :

Learning: One example is Antarctica which is a condominium governed by the Antarctica Treaty
system.

States can exercise their rights jointly, without dividing it into 'national' zones.

www.insightsias.com 67
© Insights Active Learning | All rights reserved - 78168. You may not reproduce, distribute or exploit the contents in any form without
written permission by copyright owner. Copyright infringers may face civil and criminal liability
Total Marks : 200
TEST - 10(MOCK TEST - 8) Mark Scored : 0

A major problem, and the reason so few of such territories have existed, is the difficulty of ensuring
co-operation between the sovereign powers; once the understanding fails, the status is likely to
become untenable.

Q Source: Map-based questions: Antarctica

7
2 27
0 3
5
41
-9
. com
ai l
gm
7 @
isra
i t m
p ulk
a -
i sr
M
l k it
Pu

www.insightsias.com 68
© Insights Active Learning | All rights reserved - 78168. You may not reproduce, distribute or exploit the contents in any form without
written permission by copyright owner. Copyright infringers may face civil and criminal liability

Вам также может понравиться